Download as pdf or txt
Download as pdf or txt
You are on page 1of 126

PSYCHIATRY

By: Moses Kazevu


TABLE OF CONTENTS
INTRODUCTION TO PSYCHIATRY ................................................................ 3
IMPORTANT PSYCHIATRIC TERMINOLOGY ......................................... 3
PSYCHIATRIC HISTORY AND EXAMINATION ....................................... 5
THE EGO, SUPEREGO AND ID ........................................................................ 6
THE ID (OR IT) ................................................................................................ 6
THE EGO (OR I) .............................................................................................. 7
THE SUPEREGO (OR ABOVE I) ................................................................... 8
FREUD’S STAGES OF PSYCHOSEXUAL DEVELOPMENT .................... 9
PSYCHOTIC DISORDERS ............................................................................... 12
SCHIZOPHRENIA ......................................................................................... 14
HIV PSYCHOSIS ........................................................................................... 26
DEMENTIA, DELIRIUM AND NEUROPSYCHIATRY ................................ 30
DEMENTIA .................................................................................................... 30
DELIRIUM ..................................................................................................... 35
MOOD DISORDES ............................................................................................ 40
CLASSIFICATION ........................................................................................ 40
MOOD EPISODES ......................................................................................... 41
MOOD DISORDERS ..................................................................................... 43
NEUROTIC, STRESS-RELATED AND SOMATOFORM DISORDERS ...... 56
ANXIETY DISORDER .................................................................................. 58
PERSONALITY DISORDERS .......................................................................... 71
CLASSIFICATION ........................................................................................ 72
DIAGNOSIS ................................................................................................... 82
TREATMENT ................................................................................................ 82
FACTITIOUS DISORDER (MUNCHAUSEN SYNDROME) ......................... 84
CLINICAL FEATURES ................................................................................. 84
DIAGNOSIS ................................................................................................... 86
TREATMENT AND PROGNOSIS ............................................................... 86
IMPLICATIONS ............................................................................................ 87

1
MALINGERING ................................................................................................ 87
CLINICAL FEATURES ................................................................................. 87
SEXUAL PERVERSIONS AND PARAPHILIAS ............................................ 89
ETIOLOGY .................................................................................................... 89
COMMON PARAPHILIAS ........................................................................... 90
TREATMENT ................................................................................................ 93
SUBSTANCE MISUSE...................................................................................... 95
ALCOHOL...................................................................................................... 97
REVISION ........................................................................................................ 108
CASE STUDIES ........................................................................................... 108

2
INTRODUCTION TO PSYCHIATRY
 Psychiatry is a medical specialty.
 It mostly deals with conditions in which the symptoms and signs
predominantly related to emotions, perception, thinking or memory.
 It also encompasses learning disability and the psychological aspects of the
rest of medicine.
 There are 2 primary characterizations of psychiatric disorders:
 Organic disorder: this is a form of decreased mental function due to
medical or physical disease, rather than a psychiatric illness. Examples
include dementia, and the results of encephalitis, meningitis, hypoxia,
brain injury or chronic drug abuse and alcoholism.
 Functional disorder: this is a form of decreased mental function with no
known organic basis but are believed to be the result of psychological
factors such as emotional conflicts or stress. Examples include antisocial
behaviors, phobias and personality disorders.
 The major diagnostic categories are:
 Psychoses (formerly called major mental illnesses): Schizophrenia.
 Neuroses (formerly called minor mental illnesses): anxiety disorders,
dissociative disorders, somatoform disorders, eating disorders, and sexual
dysfunction.
 Organic disorder: Dementia and Delirium.
 Mood disorders: Depression, Mania, and Bipolar disorder.
 Personality disorder.
 Substance misuse.

IMPORTANT PSYCHIATRIC TERMINOLOGY


DISTURBANCE OF PERCEPTION
 Illusion: misperception of real external sensory stimuli.
 Hallucination: false sensory perception not associated with real external
stimuli.
 Hallucinations can be auditory, visual, olfactory, gustatory, tactile, somatic
(visceral).
 Micropsia: a condition affecting human visual perception in which objects are
perceived to be smaller than they are.
 Micropsia can be caused by optical factors, by distortion of images in the
eye, by changes in the brain, and from psychological factors.
 Dissociative phenomena are linked with micropsia, which may be the
result of brain-lateralization disturbance.
 Macropsia: objects are visually perceived to be larger than they are.

3
FORMAL THOUGHT DISTURBACES
 Neologism- new word created by patient.
 Word salad- incoherent mixture of words and phrases.
 Incoherence- no logical or grammatical connection.
 Preservation- persisting response to a previous stimulus after a new stimulus
has been presented.
 Verbigeration- meaningless repetition of words/phrases.
 Loosening of association- shift without logical relation.
 Blocking- abrupt interruption of train of thought.
DISTURBANCE OF THOUGHT CONTENT
 Delusion- false beliefs held by a patient that are not consistent with patient’s
intelligence, social and cultural background and cannot be corrected by
reasoning.
 Delusions of persecution- being harassed, cheated or persecuted
 Delusions of grandeur- exaggerated conception of importance, power or
identity
 Delusions of reference- behavior of others refers to the patient
 Nihilistic delusions- self, others or the world is nonexistent
 Delusions of poverty- to be bereft of all material possessions
 Somatic delusions- involving functioning of the body
 Delusions of self-accusation- feeling of remorse and guilt
 Delusion of control- persons will, thoughts or feelings are being controlled
by external forces (thought withdrawal, thought insertion, thought
broadcasting, thought control)
 Delusion of infidelity (Othello syndrome)- delusion of infidelity of a
spouse or partner. This affects males and less often females. It is
characterized by recurrent accusations of infidelity, searches for evidence,
repeated interrogation of the partner etc.
 Erotomania- delusional belief that a person is in love
CATATONIC SYMPTOMS
 Catatonic stupor- markedly slowed motor activity often to a point of
immobility.
 Catatonic posturing- voluntary assumption of an inappropriate or bizarre
posture.
 Echopraxia- pathological imitation of movements.
 Stereotypy- habitual involuntary movement
 Command automatism-automatic following of suggestions.

4
PSYCHIATRIC HISTORY AND EXAMINATION
 Demographics and data promoting involuntary admission
 Family history
 Personal history:
 Past illnesses (psychiatric, medical, substance abuse)
 Prenatal, perinatal history and early psychomotor development
 Pre-school and school history
 Occupational history
 Marital and relationship history including sexual history
 Social history and current living situation
 Traumatic experiences
 Present illness:
 Onset, precipitating factors
 Course, periodicity
 Chief complaint and problem
 Mental status examination
 General description (orientation, appearance, behavior, attitude, speech)
 Mood and affectivity
 Perception e.g. hallucinations
 Thought process and content (e.g. formal thought disturb, delusions)
 Memory
 Judgement and insight, suicidal thoughts
 Diagnosis
 Differential diagnosis
 Further diagnostic plan (interviews with family members¸ psychological,
neurological, laboratory examination, imaging methods etc.)
 Treatment plan

5
THE EGO, SUPEREGO AND ID
 According to Freud’s model of the psyche:
 The ID is the primitive and instinctual part of the mind that contains sexual
and aggressive drives and hidden memories.
 The ego is the realistic part that mediates between the desires of the ID and
the super-ego.
 The super-ego operates as a moral conscience.
 Freud’s single most enduring and important idea was that the human psyche
(personality) has more than one aspect.
 Freud (1923) saw the psyche structured into 3 parts (i.e. tripartite), the ID, ego
and superego, all developing at different stages in our lives.
 These are systems, not parts of the brain or in any way physical.
 Each part of the personality comprises unique features, the systems interact to
form a whole. Each part of the whole makes a relative contribution to an
individual’s behavior.

THE ID (OR IT)


 The ID is the primitive and instinctive component of personality.
 It consists of all the inherited (i.e. biological) components of personality
present at birth including:
 The sex (life) instinct- Eros (which contains the libido)
 The aggressive (Death) instinct- Thanatos
 The ID is the impulsive (and unconscious) part of our psyche which responds
directly and immediately to the instincts.
 The personality of the newborn child is all ID and only later does it develop
an ego and super-ego.
 The ID remains infantile in its function throughout a person’s life and does
not change with time or experience, as it is not in touch with the external
world.
 The ID is not affected by reality, logic or the everyday world, as it operates
within the unconscious part of the mind.
 The ID operates on the pleasure principle (Freud 1920) which is the idea that
every wishful impulse should be satisfied immediately, regardless of the
consequences.
 When the ID achieves its demands, we experience pleasure when it is denied
we experience ‘unpleasure’ or tension.
 The ID engages in primary process thinking which is primitive, illogical,
irrational and fantasy oriented.

6
 This form of process thinking has no comprehension of objective reality and
is selfish and wishful in nature.

THE EGO (OR I)


 The ego is the part of the ID which has been modified by the direct influence
of the external world.
 The ego develops to mediate between the unrealistic ID and the external real
world.
 It is the decision-making component of personality.
 Like the ID, the ego seeks pleasure (i.e. tension reduction) and avoids pain,
but unlike the ID, the ego is concerned with devising a realistic strategy to
obtain pleasure.
 The ego has no concept of right or wrong, something is good simply if it
achieves its end of satisfying without causing harm to itself or the id.
 Often the ego is weak relative to the headstrong ID and the best the ego can
do is stay on, pointing the ID in the right direction and claiming some credit
at the end as if the action were its own.
 Freud made the analogy of the ID being a horse while the ego is the rider.
 The ego is ‘like a man on horseback, who has to hold in check the superior
strength of the horse’.
 Ideally, the ego works by reason, whereas the ID is chaotic and unreasonable.
 The ego operates according to the reality principle, working out realistic ways
of satisfying the ID’s demands often compromising or postponing satisfaction
to avoid negative consequences of society.
 The ego considers social realities and norms, etiquette and rules in deciding
how to behave.
 If the ego fails in its attempt to use the reality principle, and anxiety is
experienced, unconscious defense mechanisms are employed, to help ward off
unpleasant feeling (i.e. anxiety) or make good things feel better for the
individual.
 The ego engages in secondary process thinking, which is rational, realistic and
orientated towards problem-solving.
 If a plan of action does not work, then it is thought through again until a
solution is found.
 This is known as reality testing and enables the person to control their
impulses and demonstrate self-control via mastery of the ego.
 An important feature of clinical and social work is to enhance ego functioning
and help the client test reality through assisting the client to think through their
options.

7
THE SUPEREGO (OR ABOVE I)
 The superego incorporates the values and morals of society which are learned
from one’s parents and others.
 It develops around the age of 3-5 during the phallic stage of psychosexual
development.
 The superego’s function is to control the ID’s impulses, especially those which
society forbids, such as sex and aggression.
 It also has the function of persuading the ego to turn to moralistic goals rather
than simply realistic ones and to strive for perfection.
 The superego consists of 2 systems:
 The conscience: can punish the ego through causing feelings of guilt. For
example, if the ego gives in to the ID’s demands, the superego may make
the person feel bad through guilt.
 The ideal self (or ego-ideal): is an imaginary picture of how you ought to
be, and represents career aspirations, how to treat other people, and how to
behave as a member of society.
 Behavior which falls short of the ideal self may be punished by the superego
through guilt.
 The super-ego can also reward us through the ideal self when we behave
‘properly’ by making us feel proud.
 If a person’s ideal self is too high a standard, then whatever the person does
will represent failure.
 The ideal self and conscience are largely determined in childhood from
parental vales and how you were brought up and either over- or under-indulges
once he or she becomes an adult.

8
FREUD’S STAGES OF PSYCHOSEXUAL
DEVELOPMENT
 Sigmund Freud (1856-1939) is probably the most well-known theorist when
it comes to the development of personality.
 Freud’s stages of psychosexual development are, like other stage theories.
 They are completed in a predetermined sequence.
 They can result in either successful completion or a healthy personality.
 They can result in failure, leading to an unhealthy personality.
 This theory is probably the most well-known as well as the most controversial
as Freud believed that we develop through stages based upon a particular
erogenous zone.
 During each stage, an unsuccessful completion means that a child becomes
fixated on that particular erogenous.
 To remember the stages of psychosexual development, use the mnemonic:
 Old= Oral stage (birth to 18 months)
 Aged= Anal stage (18 months to 3 years)
 Parents= Phalic stage (3 to 6 years)
 Love= Latent stage (6 to 12 years i.e. puberty)
 Grapes= Genital stage (12 years onwards)
ORAL STAGE (BIRTH TO 18 MONTHS)
 During the oral stage, the child is focused on oral pleasures (sucking).
 Too much or too little gratification can result in an oral fixation or oral
personality which is evidenced by a preoccupation with oral activities.
 This type of personality may have a stronger tendency to smoke, drink alcohol,
over eat or bite his or her nails.
 Personality wise, these individuals may become overly dependent upon
others, gullible (easily deceived or cheated) and perpetual followers.
 On the other hand, they may also fight these urges and develop pessimism and
aggression toward others.
ANAL STAGE (18 MONTHS TO 3 YEARS)
 The child’s focus of pleasure in this stage is on eliminating and retaining feces.
 Through society’s pressure, mainly via parents, the child has to learn to control
anal stimulation.
 In terms of personality, after effects of an anal fixation during this stage can
result in an obsession with cleanliness, perfection and control (anal retentive).
 On the opposite end the spectrum, they may become messy and disorganized
(anal expulsive).

9
PHALLIC STAGE (3 YEARS TO 6 YEARS)
 The pleasure zone switches to the genitals.
 Freud believed that during this stage the boy develops unconscious sexual
desires for their mother.
 Because of this he becomes rivals with his father and sees him as competition
for the mother’s affection.
 During this time, boys also develop a fear that their father will punish them
for these feelings, such as by castrating them.
 This group of feelings is known as Oedipus complex (after the Greek
Mythology figure who accidentally killed his father and married his mother).
 Later it was added that girls go through a similar situation, developing
unconscious sexual attraction to their father.
 Although Freud strongly disagreed with this, it has been termed the Electra
complex by more recent psychoanalysts.
 According to Freud, out of fear of castration and due to the strong competition
of his father, boys eventually decide to identify with him rather than fight him.
 By identifying with his father, the boy develops masculine characteristics and
identifies himself as a male, and represses his sexual feelings toward his
mother.
 A fixation at this stage could result in sexual deviancies (both overindulging
and avoidance) and weak or confused sexual identity according to
psychoanalysts.
LATENCY STAGE (6 YEARS TO PUBERTY)
 It’s during this stage that sexual urges remain repressed.
 Children interact and play mostly with same sex peers.
GENITAL STAGE (PUBERTY ON)
 The final stage of psychosexual development begins at the start of puberty
when sexual urges are once again awakened.
 Through the lessons learned during the previous stages, adolescents direct
their sexual urges onto opposite sex peers.
 It is understood that the primary focus of pleasure is the genitals.

10
STAGE AGE FOCUS OF DEVELOPMENT ADULT
LIBIDO FIXATION
Oral Birth to 18 Mouth Feeding  Smoke
months  Bite-nails
 Over-eating
Anal 18 months Anus Toilet training  Orderliness (anal
to 3 years retentive)
 Messiness (anal
expulsive)
Phalic 3 to 6 years Genital Oedipus Complex  Sexual
Electra complex dysfunction
Latent 6 to 12 None Socialism None
years Development of
(puberty) skills
Genital 12 years Genital Sexual maturity Mentally health
onwards

11
PSYCHOTIC DISORDERS
 Psychosis is a condition where an individual is detached from reality.
 It denotes a disturbance in sense of reality e.g. beliefs that are not true and
seeing/hearing things that are not there.
 Individuals with psychosis often find it difficult to differentiate what is real
from what is unreal.
 Psychosis is characterized by:
 Delusions: strong false beliefs held by a person that are not consistent with
their intelligence, social and cultural background and cannot be easily
corrected by reasoning. They include:
o Delusions of reference: certain events that happen relate directly to
the patient
o Delusion of grandiose: person has a unique sense of significance
o Paranoid delusions: belief they are being harmed or watched. This
can lead to patients avoiding their medication due to paranoid
delusions about the medication
o Delusions of control: thoughts and actions are being controlled.
o Erotomania: delusion that someone is in love with them.
 Hallucination: false perception of stimulus that does not exist. They
include:
o Auditory hallucinations (most common)
o Visual hallucinations (common)
o Tactile hallucinations
o Olfactory hallucinations
o Gustatory hallucinations
o Visceral hallucinations
 Disorganized behavior: can be directly observed
 Disorganized thinking: can be indirectly observed i.e. through speech
patterns e.g.
o Poverty of content/speech (Alogia)
o Tangential speech- getting off topic
o Thought blocking
o Word salad
o Preservation- words and ideas repeated even after topic has been
switched
 Agitation and aggression: Anxiety, heightened emotions, heightened motor
activity.

12
 Underlying causes include:
 Psychiatric conditions e.g. schizophrenia
 Medical conditions e.g. delirium
 Substance abuse e.g. alcohol and hallucinogens (e.g. LSD)
 Medication e.g. antiparkinson drugs (levo-dopa) and antivirals

13
SCHIZOPHRENIA
 Schizophrenia (Splitting of the mind) is a psychosis, typically presenting in
young adults (16 to 25 years) affecting 1% of the population.
 Even though schizophrenia can be interpreted to mean splitting of the
mind, it does not refer to a split personality. It rather refers to a scattered
or fragmented pattern of thinking.
 It is distinguished from other psychoses by:
 The presence of specific types of delusions, hallucinations and thought
disorder.
 The primary disorder, which is not one of affective (mood) or organic
etiology.
 The clinical course.
 Schizophrenia is characterized by:
 A collection of abnormalities in thinking, behavior and emotion.
 A group of characteristic positive and negative symptoms (it is a
syndrome)
 Deterioration in social, occupational or interpersonal relationships.
 Continuous signs of the disturbance for at least 6 months.
 There is no single symptom that is pathognomonic and there is a wide
variation in clinical presentation.
 Schizophrenia is prevalent across racial, sociocultural and national
boundaries, with a few exceptions in the prevalence rates in some isolated
communities.
 Men and women are equally affected but have different presentations and
outcomes:
 Men tend to present in early to mid-20s
 Women present in late 20s
 Men tend to have more negative symptoms and poorer outcome compared
to women.
 Some studies suggest this could be due to estrogen regulation of dopamine.
 Schizophrenia rarely presents before age 15 or after age 55.
 There is strong genetic predisposition:
 50% concordance rate among monozygotic twins
 40% risk of inheritance if both parents have schizophrenia
 12% risk if one first-degree relative is affected
 Substance use is comorbid in many patients with schizophrenia. The most
commonly abused substance is nicotine (>50%), followed by alcohol,
cannabis and cocaine.

14
 Post-psychotic depression is the phenomenon of schizophrenic patients
developing a major depressive episode after resolution of their psychotic
symptoms.
HISTORY
 Emil Kraeplin: recognized that this illness develops relatively early in life and
its course is likely deteriorating and chronic but was not followed by any
organic changes of the brain, detectable at that time. He therefore called it
‘Dementia praecox’ (Dementia: deterioration; praecox: early onset). He
recognized the characteristic features of dementia praecox such as delusions,
hallucinations, disturbances of affect and motor disturbances.
 Eugen Bleuler (1911): renamed dementia praecox as schizophrenia (splitting
of the mind). He recognized the cognitive impairment in this illness. Bleuler
described the characteristic symptoms which were then thought to be
diagnostic of schizophrenia, these included (4As of Bleuler):
o Affective blunting/disturbance: disturbances of affect such as
inappropriate affect
o Association disturbances: loosening of association, thought disorder,
fragmented thinking
o Autism: withdrawal from reality
o Ambivalence: marked inability to decide for or against (fragmented
emotional response)
According to Bleuler, these groups of symptoms are “primary” for the
diagnosis of schizophrenia. The other known symptoms of schizophrenia
include hallucinations, delusions which appeared in schizophrenia very often.
He called them as “secondary symptoms”, because they could be seen in any
other psychotic disease, which are caused by quite different factors- from
intoxication to infection or other disease entities.
 Kurt Schneider: Emphasized the role of psychotic symptoms, as
hallucinations, delusions and gave them the privilege of “first rank symptoms”
even in the concept of the diagnosis of schizophrenia. First rank symptoms
include:
 Delusions:
o Delusion of perception
o Delusion of control
o Delusion of influence and passivity (passivity of thought, feelings or
actions)

15
 Hallucinations
o Third person auditory hallucinations
o Voices commenting on one’s action
o Running commentary (voices commenting or discussion the patient
in the third person)
 Disorders of thought process and content
o Thought echo (voices speaking out thoughts aloud)/ Echo delapance.
o Thought withdrawal: thoughts cease and subject experiences them
as removed by an external force.
o Thought insertion: experience of thoughts imposed by some external
force on person’s passive mind.
o Thought broadcasting/ diffusion: experience of thoughts escaping
the confines of self and as being experienced by others around.
CLINICAL FEATURES
 In general, the symptoms of schizophrenia are broken up into 3 categories:
 Positive symptoms:
o Hallucinations: Auditory hallucination- in third person, visual
hallucinations.
o Delusions of control- an outside force is controlling their actions and of
reference- they think insignificant remarks are directed at them like a
news caster speaking directly to them through the TV
o Disorganized/Bizarre behavior (silly behavior) e.g. wearing multiple
layers of jackets on a hot day
o Disorganized speech: word salad
o Positive formal thought disorder: autistic thinking, loosening of
association, incoherence of speech, thought block, thought withdrawal,
thought insertion, thought broadcasting, neologisms and word salad.
o Catatonic behavior: either by resistance to movement or stupor.
o These tend to respond more robustly to antipsychotic medications.
 Negative symptoms:
o Flat or blunted affect,
o Anhedonia (inability to experience pleasure),
o Alogia (Poverty of speech- lack of content in speech),
o Avolition/Apathy (decreased motivation) and lack of interest in
socialization.
o These symptoms are comparatively more often treatment resistant and
contribute significantly to the social isolation of schizophrenic patients

16
 Cognitive symptoms:
o Impairments in attention, executive function and working memory.
These symptoms may lead to poor work and school performance. They
are often subtle and difficult to notice.

Think of positive symptoms as things that are ADDED onto


normal behavior. Think of negative symptoms as things that
are SUBTRACTED or missing from normal behavior.

 The symptoms of schizophrenia often present in 3 phases:


1. Prodromal: decline in function that precedes the first psychotic episode.
The patient may become socially withdrawn and irritable. He or she may
have physical complaints, declining school/work performance, and/or
new found interest in religion or the occult.
2. Psychotic: perceptual disturbances, delusions and disordered thought
process/content.
3. Residual: Occurs following an episode of active psychosis. It is marked
by mild hallucinations or delusions, social withdrawal and negative
symptoms.
ETIOLOGY
 Cause is unknown.
 Increased levels of dopamine have been implicated.
 Other implicated neurotransmitters include norepinephrine, serotonin, GABA,
and glutamate.
 Twin studies support a genetic basis however no genes have been conclusively
linked to schizophrenia yet.
 Environmental factors e.g. early/prenatal infection and autoimmune disorders
e.g. celiac disease have been linked to schizophrenia.
PATHOPHYSIOLOGY OF SCHIZOPHRENIA: THE
DOPAMINE HYPOTHESIS
 Though the exact cause of schizophrenia is not known, it appears to be partly
related to increase dopamine activity in certain neuronal tracts.
 Evidence to support this hypothesis is that most antipsychotics successful in
treating schizophrenia are dopamine receptor antagonists.
 In addition, cocaine and amphetamines increase dopamine activity and can
lead to schizophrenic-like symptoms.

17
 The theorized dopamine pathways affected in schizophrenia include:
 Mesolimbic: Excessive dopaminaergic activity responsible for positive
symptoms.
 Prefrontal cortical: inadequate dopaminergic activity responsible for
negative symptoms.
 Other important dopamine pathways affected by antipsychotics:
 Tuberoinfundibular: blocked by antipsychotics causing
hyperprolactinemia which may lead to gynecomastia, galactorrhea, sexual
dysfunction and menstrual irregularities.
 Nigrostriatal: Blocked by antipsychotics, causing
Parkinsonism/extrapyramidal side effects such as tremor, rigidity, slurred
speech, akathisia (unpleasant, subjective sense of restlessness and need to
move often manifested by the inability to sit still), dystonia and other
abnormal movements.
 Other neurotransmitter abnormalities implicated in schizophrenia include:
 Elevated serotonin: some of the second-generation (atypical)
antipsychotics (e.g. risperidone and clozapine) antagonize serotonin and
weakly antagonize dopamine.
 Elevated norepinephrine: long-term use of antipsychotics has been shown
to decrease activity of noradrenergic neurons.
 Decreased gamma-aminobutyric acid (GABA): there is a decrease in
expression of the enzyme necessary to create GABA in the hippocampus
of schizophrenic patients.
 Decreased levels of glutamate receptors: schizophrenic patients have fewer
NMDA receptors. This corresponds to the psychotic symptoms observed
with NMDA antagonists like ketamine.
DIAGNOSIS
 For the diagnosis of schizophrenia, it is necessary for either:
 Presence of one very clear symptom- from (a) to (d) for one month or more
 Presence of at least 2 symptoms - from point (e) to (h) for one month or
more
 Symptoms include
a. The hearing of own thoughts, the feelings of thought withdrawal, thought
insertion or thought broadcasting
b. The delusions of control, outside manipulation and influence, or the feeling
of passivity which are connected with the movements of the body or
extremities, specific thoughts, acting or feelings, delusional perception

18
c. Hallucinated voices, which are commenting pertaining the behavior of the
patient or they talk about him between themselves or the other types of
hallucinatory voices, coming from different parts of the body.
d. Permanent delusions of different kind, which are inappropriate and
unacceptable in the given culture
e. The lasting hallucination of every form
f. Blocks or intrusion of thoughts into the flow of thinking and resulting
incoherence and irrelevance of speech or neologisms
g. Catatonic behavior
h. Negative symptoms, for instance the expressed apathy, poor speech,
blunting and inappropriateness of emotional reactions
i. Expressed and conspicuous qualitative changes in patient’s behavior, the
loss of interests, hobbies, aimlessness, inactivity, loss of relations to others
and social withdrawal.
 According to DSM-IV on diagnosis of schizophrenia is that symptoms of
schizophrenia should last 6 consecutive months (with at least 1 month of the
active phase). If symptoms are present between 1-6 month a diagnosis of
schizophreniform is made.
 70% of patients with schizophreniform are later diagnosed to have
schizophrenia.
CLINICAL SUBTYPES
PARANOID SCHIZOPHRENIA (F20.0)
 Characterized mainly by delusions of persecution, feelings of passive or active
control, feeling of intrusion and often by megalomanic tendencies also.
 The delusions are not usually systemized too much, without tight logical
connections and are often combined with hallucinations of different senses,
mostly with hearing voices.
 Disturbances of affect, volition and speech and catatonic symptoms are either
absent or relatively inconspicuous.
HEBEPHRENIC/ DISORGANIZED SCHIZOPHRENIA (F20.1)
 It is characterized by disorganized thinking with blunted and inappropriate
emotions.
 It begins mostly in adolescent age; the behavior is often bizarre. There could
appear mannerisms, grimacing, inappropriate laugh and joking,
pseudophilosophilcal brooding and sudden impulsive reactions without
external stimulation.
 Usually the prognosis is poor because of the rapid development of negative
symptoms, particularly flattening of affect and loss of volition.

19
 Hebephrenia should normally be diagnosed only in adolescents or young
adults.
CATANTONIC SCHIZOPHRENIA (F20.2)
 It is characterized mainly by motoric activity which might be strongly
increased (hyperkinesis) or decreased (stupor) or automatic obedience and
negativism.
 We recognize two forms:
 Productive form: which show catatonic excitement, extreme and often
aggressive activity. Treatment by neuroleptics or by electroconvulsive
therapy
 Stuporose form: characterized by general inhibition of patient’s behavior
or at least by retardation and slowness, followed often by mutism,
negativism, flexibilitas cerea or by stupor. The consciousness is not absent.
UNDIFFERENTIATED SCHIZOPHRENIA (F20.3)
 Psychotic conditions meeting the general diagnostic criteria for schizophrenia
but not conforming to any of the subtypes in F20.0 – F20.2 or exhibiting the
features of more than one of them without a clear predominance of a particular
set of diagnostic characteristics.
 This subgroup represents also the former diagnosis of atypical schizophrenia.

Figure 1: Subtypes of acute schizophrenia

20
OTHER CONSIDERATIONS
POSTSCHIZOPHRNIC DEPRESSION (F20.4)
 A depressive episode, which may be prolonged, arising in the aftermath of a
schizophrenic illness.
 Some schizophrenic symptoms either positive or negative must still be present
but they no longer dominate the clinical picture.
 These depressive states are associated with an increased risk of suicide.
RESIDUAL SCHIZOPHRENIA (F20.5)
 A Chronic stage in the development of schizophrenia with clear succession
from the initial stage with one or more episodes characterized by general
criteria of schizophrenia to the late stage with long-lasting negative symptoms
and deterioration (not necessarily irreversible).
SIMPLE SCHIZOPHRENIA (F20.6)
 Simple schizophrenia is characterized by early and slowly developing initial
stage with growing social isolation, withdrawal, small activity, passivity,
avolition and dependence on the others.
 The patients are indifferent, without any initiative and volition. There is not
expressed the presence of hallucinations and delusions.
PERSISTENT DELUSIONAL DISORDERS (F22)
 Includes a variety of disorders in which long-standing delusions constitute the
only, or the most conspicuous, clinical characteristic and which cannot be
classified as organic, schizophrenic or affective.
 Their origin is probably heterogenous but it seems that there is some relation
to schizophrenia.
DELUSIONAL DISORDER (F22.0)
 A disorder characterized by the development of one delusion or of the group
of similar related delusions, which are persisting unusually long, very often
for the whole life.
 Other psychopathological symptoms- hallucinations, intrusion of thoughts etc.
are not present and are excluding this diagnosis.
 It begins usually in the middle age.

21
ACUTE AND TRANSIENT PSYCHOTIC DISORDERS
 The criteria should be the following features:
 Acute beginning (to two weeks)
 Presence of typical symptoms (quickly changing “polymorphic
symptoms”)
 Presence of typical schizophrenic symptoms
 Complete recovery usually occurs within a few months often within a few
weeks or even days.
 The disorder may or may not be associated with acute stress, defined as
usually stressful events preceding the onset by one to two weeks.

INVESTIGATIONS
 Assessment includes
 Full history (objective/collateral and subjective)
 Mental state examination
 Neurological examination
 Investigate differentials (causes) if indicated:
 Blood investigations:
o Endocrine: Thyroid function test (hyper/hypothyroidism), Cortisol
(Addison/Cushing disease)
o Calcium (Hyper/hypocalcemia)
o Syphilis & serology
o Supportive: FBC, U&E and LFTs (for management of drugs mostly)
 Urine drug screen: to rule out substance induced psychosis from intoxication
or withdrawal (some implicated drugs antiparkisonian agents,
anticonvulsants, antihistamines, antimicrobials, anticholinergics,
antihypertensives, NSAIDs, alcohol, cocaine, hallucinogens- [LSD, Ecstasy],
Cannabis, Benzodiazepines, barbiturates, PCP, inhalants)
 Imaging:
o MRI/CT: to rule out CNS disease (multiple sclerosis, Tumors/space
occupying lesions), cerebrovascular disease, Alzheimer’s disease,
Parkinson’s disease, encephalitis, prion disease.
o EEG: epilepsy (often temporal lobe)

22
TREATMENT
 Non-pharmacological therapy: Psychotherapy/talk therapy
o Cognitive behavioral therapy: 16 sessions, focusing on re-evaluating
abnormal thoughts and perceptions and reducing the distress resulting
symptom.
o Family therapy: at least 10 sessions over 3-12 months
 Psychoeducation: e.g. how to respond to patient’s delusions,
advice on crisis management and emphasizing the importance of
creating low stress environments at home.
 ART therapy: is another option. It can help with self-expression
and is delivered in grouped thus alleviating social isolation.

 Pharmacological therapy:
o 1st line: 1st generation or 2nd generation antipsychotic (D2 antagonist)
 1st generation (typical):
 Chlorpromazine initially 25 mg 3 times a day orally, adjust
according to response, alternatively 75mg once daily,
adjusted according to response, dose to be taken at night,
maintenance 75-300mg daily, increased if necessary up to
1g daily, this dose may be required in psychoses.
 25-50mg every 6-8 hours IM for acute psychoses
 2 generation (atypical): Clozapine
nd

 12.5 mg 1-2 times a day for day 1, then 25-50mg for day
2, then increased if tolerated in steps of 25-50 mg daily
dose to be increased gradually over 14-21 days, increased
to up to 300mg daily in divided doses, larger dose to taken
at night, up to 200mg daily may be taken as a single dose
at bedtime, increased in steps of 50-100mg 1-2 times a
week if required, it is preferable to increase once a week,
usual dose 200-450 mg daily, max 900mg per day, if
restarting after interval of more than 48 hours, 12.5mg
once or twice on first day (but may be feasible to increase
more quickly than on initiation)- extreme caution if
previous respiratory or cardiac arrest with initial dosing.
o 2 line: Clozapine. Offer if 2 different antipsychotics were ineffective
nd

o Start low and titrate up, then observe effectiveness for 4-6 weeks at
optimum dose.

23
 The acute psychotic schizophrenic patients will respond usually to
antipsychotic medication.
 According to current consensus we use the first line therapy the newer atypical
antipsychotics, because their use is not complicated by appearance of
extrapyramidal side-effects or these are much lower than with classical
antipsychotics.

 Side effects from dopamine blockade:


 Dyskinesis
o Acute extrapyramidal symptoms:
 Pseudoparkinsonism (drug induced parkinsonism)- can be seen
within hours of initiating drug: resting tremors, bradykinesia,
rigidity
 Dystonia (Severe muscle rigidity with pain)- Torticollis
 Akathisia (unpleasant, subjective sense of restlessness and need to
move often manifested by the inability to sit still)
 Management: antimuscarinic drugs
o Chronic extrapyramidal symptoms:
 Tardive dyskinesia (upregulation of dopamine receptors)
hyperkinetic disorder with similar features to Huntington disease
 Management: discontinuation/switch to atypical antipsychotics
 Dysphoria: blocked of receptor in mesolimbic system (worsening negative
symptoms of schizophrenia)
 Endocrine dysfunction: hyperprolactinemia- gynecomastia, amenorrhea,
weight gain, malignant hyperthermia (neuroleptic malignant syndrome)-
blockage of dopamine receptors, increased muscle rigidity (increased
electron transport change)
 Side effects of muscarinic blockade: convulsion, coma and torsades
 Side effects from alpha blockade: hypotension and reflex tachycardia

24
 During course of treatment, monitor:
 Weight: weekly for first 6 weeks, then at 3 months.
 Blood parameters such as full blood count and blood glucose at 3 months
as well as prolactin levels at 6 months.
 Electrocardiograph monitoring if haloperidol or pimozide are used.
 Note: if health deteriorates treatment should be further titrated and continued
in hospital.

A 24-year- old male graduate student without prior medical or psychiatric


history is reported by his mother to have been very anxious over the past 9
months, with increasing concern that people are watching him. He now
claims to “hear voices” telling him what must be done to “fix the country”.

Question: what are the important workup?


Answer:
 Thyroid-stimulating hormone (TSH)
 Rapid plasma reagin (RPR)
 Brain imaging
Question: What is the likely diagnosis?
Answer: Schizophrenia
Question: What is the next step?
Answer: Antipsychotics

25
HIV PSYCHOSIS
 HIV is the most common infectious agent known to cause cognitive
impairment.
 Psychosis is one manifestation of HIV-associated dementia.
 It tends to be more common in people who have a history of prior psychiatric
disorder or drug abuse (especially amphetamines and other stimulants).
 Psychosis is sometimes also a sign of very advanced AIDS, and patients
should seek treatment immediately.
 Psychosis is a generic term for any one of a number of symptomatic
manifestations of thought disorders.
 Psychotic symptoms can be part of a severe major depressive syndrome,
schizophrenia, mania in bipolar disorder, or extremes of obsessive-compulsive
disorder.
 Some of the symptoms of psychotic episode include: Hallucinations,
Delusions, Paranoia, Mania and Depression.
 Depression and HIV can cause:
 Loss of sexual desire and function
 Lack of appetite
 Headache
 Insomnia
 Fatigue
 Upset stomach
 Diarrhea
 Restlessness or anxiety
CLINICAL FEATURES
 Acute HIV symptoms are similar to those of other viral infections. They
include: Tiredness, Headache, Weight loss, Frequent fever, Sweats, Lymph
node enlargement, and Rash
 HIV-associated neurocognitive disorder:
 Clinical experience and research provide substantial evidence that HIV
directly infects the brain soon after initial infection.
 It can result in organic disease expression such as central nervous system
impairment, dementia, pain and mood disorders.
 Clinical manifestation varies depending on the part(s) of the brain affected.
 Decline may be observed in executive functioning, attention, working
memory and psychomotor activity.
 Psychiatric and neuromotor symptoms may also be present.

26
PSYCHIATRIC ILLNESS AND HIV/AIDS
 There is a strong link between mental illness and HIV/AIDS.
 50% or more of patients with HIV/AIDS have a comorbid psychiatric
disorder.
 The prevalence of mental illness in patients with HIV/AIDS is reported to be
8 times higher than in those without HIV/AIDS.
 Depression, bipolar disorder, anxiety disorders, delirium, substance abuse and
schizophrenia have all been identified in persons receiving highly active
antiretroviral therapy (HAART).
 Patients with HIV/AIDS and psychiatric illness have a decreased quality of
life, poor adherence to medications, faster disease progression and increased
mortality.

DIAGNOSIS
 Patients whose diagnoses is schizophrenia, schizoaffective disorder, and
bipolar disorder are at greater risk for HIV infection.
 Patients with HIV/AIDS with primary psychosis may have poor medication
adherence rates due to illness-related confusion or paranoia about
medications.
 Furthermore, they may lack the resources to manage the complications and
stress related to living with HIV/AIDS.
 New onset/secondary psychosis has been reported in individuals with late-
stage HIV/AIDS with CD4 counts <200 who have not been diagnosed with a
psychotic disorder previously.
 These patients may experience more persecutory and grandiose delusions
rather than hallucinations.
 Neuropsychiatric symptoms in patients with HIV/AIDS may be due to the
presence of HIV or other infections in the CNS, tumors, or other inflammatory
illnesses.
 Medications that have been implicated in neuropsychiatric symptoms include
Efavirenz, rilpivirine, and other HAART regimens, interferon,
metoclopramide, corticosteroids, muscle relaxants and clonidine.
 It is possible that symptoms may continue even after medications are
discontinued, has been reported in individuals with late-stage HIV/AIDS with
CD4 counts <200 who have not been diagnosed with a psychotic disorder
previously. These patients may experience more persecutory and grandiose
delusions rather than hallucinations.

27
TREATMENT AND EFFECTS
 Antipsychotics (typical antipsychotics e.g. haloperidol, chlorpromazine)
remain the treatment of choice for psychosis in HIV/AIDS, regardless of the
cause of the symptoms.
 Chlorpromazine initially 25 mg 3 times a day orally, adjust according to
response, alternatively 75mg once daily, adjusted according to response,
dose to be taken at night, maintenance 75-300mg daily, increased if
necessary up to 1g daily, this dose may be required in psychoses.
 25-50mg every 6-8 hours IM for acute psychoses.
 Antipsychotics are usually effective in relieving symptoms of psychosis in the
short term.
 The long-term use of antipsychotics is associated with adverse effects such as
involuntary movement disorders, gynecomastia (swelling of the breast tissue
in boys or men), impotence, weight gain and metabolic syndrome.
 First generation antipsychotics (typical antipsychotics) are used.
 Many factors must be taken into consideration when choosing an
antipsychotic medication.
 Worthy of note are Drug-drug interactions (DDIs), adverse effect profiles,
patient history of antipsychotic use, cost, and patient preference.
 Psychostimulants target fatigue, apathy and psychomotor retardation.
 Tricyclics have more side effects than the selective serotonin reuptake
inhibitors (SSRIs).
 SSRIs are the most commonly prescribed antidepressants.
 Sertraline initially 50mg daily, then increased in steps of 50mg at intervals
of at least 1 week if required, maintenance 50mg daily, maximum 200mg
per day.
 They can ease symptoms of moderate to severe depression
 Relatively safe and typically cause fewer side effects than other types of
antidepressants do.
 SSRIs ease depression by increasing levels of serotonin in the brain.
o Serotonin is often referred to as the “feel good hormone”.
o It carries messages between brain cells and contributes to well-being,
good mood, appetite, social behavior. It as well helps to regulate the
body’s sleep-wake cycle and the internal clock.
 They can also cause sedation, constipation

28
COMPLICATIONS
 Self-harm also known as self-injury is defined as the intentional, direct
injuring of body tissue done without suicidal intentions.
 Other terms such as cutting and self-mutilation have been used for any self-
harming behavior regardless of suicidal intent.
 If left untreated:
 Psychiatric illness in patients with HIV/AIDS may lead to further
transmission of HIV
 Psychiatric illness in patient with HIV/AIDS patients will engage in high
risk behaviors
 Psychiatric illness in patients with HIV/AIDS will deteriorate and lead to
poor adherence to HAART.

29
DEMENTIA, DELIRIUM AND
NEUROPSYCHIATRY
 Psychiatric disorders labelled ‘organic’ are those with demonstrable pathology
or etiology or that arise directly from a medical disorder.
 The major organic disorders are dementia and delirium.

DEMENTIA
 Dementia typically refers to a gradual decline of mental functioning,
characterized by memory problems and personality and behavioral changes.
 Dementia is commoner in women.
 Dementia is most common beyond the age of 65 years.
 Types of dementia include:
 Alzheimer’s disease (most common-80%)
 Vascular dementia
 Lewy body dementia
 Frontotemporal dementia
 Mixed
ALZHEIMER’S DISEASE
 This is the most common cause of dementia (60-80%)
 It is neurodegenerative disorder due to brain amyloidosis (which form
plaques-abnormal protein fragments between neurons) and Tau pathology
(Tau proteins are abnormally accumulated in neuron cells as tangles):
 Mis-cleavage of the amyloid precursor protein by beta and gamma
secretase instead of the usual alpha-secretase leading to the formation of
beta-amyloid (also known as A).
 Adeposits in the extracellular space and aggregates as senile plaques,
occupying up 20 % of the brain but potentially leading to lower levels in
the CSF.
 Also involves neurofibrillary tangles of Tau proteins in both the intra- and
extra-cellular spaces.
 The pattern of presentation is insidious in onset. There is progressive and
global cognitive deterioration.
 It is characterized by the 4A:
 Amnesia: recall is impaired first, with recognition initially intact. Short-
term affected before long-term, with difficulty learning new things.
 Aphasia: expressive first, with word-finding difficulties is common.
 Agnosia: e.g. difficulty naming object in hand with eyes closed. Also
anosognosia (poor insight)

30
 Apraxia: impaired motor planning skills e.g. dressing apraxia.
 Individuals also have visuospatial problems (easily get lost), affective and
psychotic symptoms and reduced executive function, apathy.
 Risk factors for Alzheimer’s:
 Age: increase with age (65-80 years)
 Family history
 Specific mutations: Apolipoprotein E-E4 (ApoE e4), CL1, CLU, PICALM
 Down’s syndrome
 Vascular risk factors
 Limited intellectual activity or stimulation
 Depression
 Traumatic brain injury, especially if later in life
VASCULAR DEMENTIA
 Due to cumulative effects of many small strokes.
 Can co-occur with Alzheimer’s but rarely with Lewy body dementia.
 Presentation is stepwise, with rapid decline in specific functions. However,
may fluctuate with partial recovery.
 There may be focal neurological signs.
 Relatively preserved personality and insight.
 Risk factors for vascular dementia:
 Ischemic heart disease and its risk factors
 Diabetes
LEWY BODY DEMENTIA
 Characterized by aggregations of Lewy bodies in substantia nigra and other
brain structures.
 Lewy bodies consist of alpha synuclein, ubiquitin and other proteins.
 It is characterized by fluctuating changes in cognition. Changes can be day to
day and cycles get progressively shorter (e.g. hours). Lucid periods may be
very distressing due to insight, leading to depression.
 Visual hallucinations: vivid and often frightening. May include children,
animals or Lilliputian hallucinations. No auditory hallucinations, so figures
don’t speak back.
 Parkinsonism including falls.
 There may also be REM sleep disorder.

31
FRONTOTEMPORAL DEMENTIA
 Frontotemporal dementia/ Pick’s disease/Frontal lobe dementia is
Characterized by atrophy of the frontal and temporal lobes.
 3 pathological subtypes based on the types of abnormal protein deposition:
 FTD: Fused in sarcoma (FUS) protein
 Phosphorylated Tau
 Transactive response DNA-binding protein 43 (TDP43)
 Frontotemporal dementia may be part of motor neuron disease.
 It usually has a younger onset (45-70)
 20% have autosomal dominant inheritance.
 Relative to Alzheimer’s more personality and speech problems early on with
episodic and topographical memory retained until quite late.
 3 main clinical syndromes:
 Behavioral variant FTD: impaired interpersonal and executive skills
including apathy, disinhibition, obsessions and stereotypies.
 Primary progressive aphasia which has 2 subtypes:
o Progressive non fluent aphasia: involving poor speech production and
o Semantic aphasia involving difficulty understanding and remembering
words and their meanings.
MANAGEMENT
 Management is multidisciplinary. Refer to specialist clinic for diagnosis. This
involves:
 Bloods and imaging
 Formal cognitive assessment e.g. with Addenbrooke’s Cognitive
Examination-Revised (ACE-R). Scores <45 suggest cognitive impairment.
 Detailed history from the patient and a collateral history
 Investigations to rule out other causes
 Blood: FBC, U&E, LFTs, Calcium, Thyroid function tests, Vitamin B12
serum assay, folate, ferritin and ceruloplasmin.
 Urine dip for UTI, especially in an acute context
 Tissue diagnosis:
o CSF tau and beta-amyloid testing can support diagnosis of Alzheimer’s
if unclear clinically.
o A definitive pathological diagnosis can only be made post-mortem but
is rarely necessary
 Imaging
o CT head:
 To look for vascular disease and rule out space occupying lesions.

32
 Small vascular changes are common but need to be significant to
support diagnosis of vascular dementia.
 Alzheimer’s may show mesial temporal lobe atrophy.
 MRI if vascular dementia is suspected.
 Medical management:
 Oral cholinesterase inhibitors are the first line treatment for mild to
moderate Alzheimer’s
o Drugs: Donepezil, galantamine or rivastigmine (also available as a
patch)
o Can be used in LBD but not in vascular dementia or FTD.
o Side effects: diarrhea and vomiting, cramps, urinary incontinence,
headache, dizziness, insomnia, exacerbate peptic ulcer disease,
bradycardia and AV block.
o Note:
 ECG is done first, as 2nd degree heart block is a contraindication.
 Check pulse after starting for bradycardia
 Take after eating due to GI side effects
 Can switch to rivastigmine patch if GI side effects especially bad.
 Memantine is a glutamate receptor antagonist used in:
o Severe Alzheimer’s
o Moderate Alzheimer’s or LBD where cholinesterase inhibitors are not
tolerated or are ineffective (having tried 2).
 Other options for behavioral and psychological symptoms:
 First optimize non-pharmacological methods and rule out underlying
causes of distress e.g. pain.
 Antipsychotics at lowest dose and duration possible only if risk to self or
others or experiencing distressing hallucinations or delusions.
 Antidepressants only in severe depression.
PROGNOSIS
 Patients with Alzheimer’s and LBD usually die within 7 years of diagnosis.
Sooner in vascular dementia.
DIFFERENTIAL DIAGNOSIS
 Reversible dementias:
 Hypothyroidism
 Electrolyte abnormalities
 Decreased Vitamin B2
 Anemia

33
 Normal pressure hydrocephalus. Classic triad of dementia, urinary
incontinence and gait ataxia
 Non-reversible dementias:
 Cruetzfeldt-Jakob’s disease (mad cow disease): a disease of the nervous
system caused by slow-acting prion which eventually affect the brain.
 Hungtington’s
 Delirium: an acutely altered cognitive state which should resolve with
treatment of the underlying cause. May co-exist with dementia.
 Depression:
 In the elderly, often features pseudodementia i.e. subjective memory loss
 Unlike Alzheimer’s, cognitive impairment will be patchy and not global.
 Worse in the morning, while Alzheimer’s is worse in the evening.
 Mild cognitive impairment
 Mild memory loss but no functional impairment
 50% progress to dementia in 5 years, while the remainder are stable,
improve or have a non-dementia pathology.

34
DELIRIUM
 Delirium is a medical emergency. It may be the only early manifestation of
serious illness.
 Think of delirium as acute brain failure- a medical emergency like other
acute organ failures.
 Terms commonly used for delirium include toxic or metabolic
encephalopathy, acute organic brain syndrome, acute confusional state,
acute psychosis, and ICU
psychosis.
 It is reversible but can
potentially advance to coma,
seizures or death.
 Delirium often goes
unrecognized.
 Almost any medical condition
can cause delirium. Delirium
can also be caused by
pharmacological drugs e.g.
drugs with anticholinergic
effects (e.g. tricyclic
antidepressants) or sedatives.
 The most common
precipitants of delirium in
children are febrile illnesses
and medications.
 Delirium is however much
more common in older
individuals than younger ones.

35
 Risk factors:
 Polypharmacy, including the use of psychotropic medications (especially
benzodiazepines and anticholinergic drugs)
 Advanced age
 Male gender
 Alcohol use (delirium tremens)
 Preexisting cognitive impairment or depression
 Prior history of delirium
 Severe or terminal illnesses
 Multiple medical comorbidities (dementia, constipation, pneumonia,
urinary tract infection)
 Impaired mobility
 Hearing or vision impairment
 Malnutrition
 Pain

36
Figure 2: The cause of delirium is unknown but theories suggest that (1) there is a disturbance in the
overall level of the neurotransmitters acetylcholine, dopamine, norepinephrine and glutamate, (2) a
problem in neuron action potential transmission (3) production of cytokines as a result of an
inflammatory process.

CLINICAL FEATURES
 Primarily a disorder of attention and awareness (i.e. orientation in time, place
and person-disorientation).
 Cognitive deficits develop acutely over hours to days.
 Symptoms fluctuate throughout the course of a day, typically worsening at
night.
 Other features include deficits in recent memory, concentration, language
abnormalities, incoherent speech, perceptual disturbances (usually visual such
as illusions or hallucinations), transient persecutory delusions, irritability and
agitation or somnolence and decreased activity (sluggish & drowsy, less
reactive & sullen as well as looking withdrawn).
 Circadian rhythm disruption and emotional symptoms are common.
 3 types of delirium are seen based on psychomotor activity:
 Mixed type
o Psychomotor activity may remain stable at baseline or fluctuate rapidly
between hyperactivity and hypoactivity.
o Most common type.
 Hypoactive (“quiet”) type
o Decreased psychomotor activity, ranging from drowsiness to lethargy
to stupor
o More likely to go undetected
o More common in the elderly

37
 Hyperactive type (“ICU psychosis”)
o Manifests with agitation/aggressive behavior, incoherent speech,
disorganized thoughts, delusions, hallucinations and disorientation.
o Less common, but more easily identified due to its disruptiveness
o More common in drug withdrawal or toxicity
 Complete recovery occurs in most hospitalized patients within about 1 week,
however, some cognitive deficits can persist for months or even remain
indefinitely.
 Note: if a patient presents with altered mental status, disorientation, confusion,
agitation or new-onset psychotic symptoms suspect delirium.
DIAGNOSIS
 Disturbance in attention and awareness
 Disturbance in an additional cognitive domain
 Develops acutely over hours to days, represents a change from baseline and
tends to fluctuate
 Not better accounted for by another neurocognitive disorder
 Not occurring during a coma
 Evidence from history, physical or labs that the disturbance is a direct
consequence of another medical condition, substance intoxication/withdrawal,
exposure to toxin or due to multiple etiologies.
DELIIUM DEMENTIA
Onset Symptoms start suddenly Slow mental decline over
(Acute)-hours/days months-years (insidious)
Short-term course Fluctuating Constant
Attention Poor Good
Symptoms Temporary resolve when Early symptoms: alert,
cause is addressed oriented, normal behavior
(hours/days, weeks to and no hallucinations
months). Delusions and
hallucinations are common,
simple and fleeting
Reversible Yes No

 Once delirium has been diagnosed the cause should be sought:


 Blood glucose, pulse-oximetry, electrocardiography and arterial blood
gases.
 Metabolic panel, full blood count with differential, urinalysis and urine
culture

38
 Urine drug screen, blood alcohol level, hepatic panel, thyroid function
tests, or chest X-ray depending on clinical presentation
 Lumbar puncture, Head imaging (heat CT or MRI brain), and EEG if focal
neurological deficits are present or a cause of delirium cannot be identified
with the initial workup.
TREATMENT
 Treat the underlying cause(s)
 Address potential exacerbating factors such as mobility limitations, sensory
deficits, sleep cycle disruption, constipation, urinary retention, dehydration
and electrolyte abnormalities, uncontrolled pain and use of unnecessary
medications.
 Encourage a family member to stay at bedside to provide company and
redirection as needed.
 Adequate supervision
 Reorient the patient on a regular basis by drawing attention to time, place and
situation and by keeping whiteboards, calendars and clocks in plain sight
 Antipsychotics (D2 antagonist) are indicated for treatment of agitation that
places the patient or others at risk.
 Haloperidol is the preferred agent and can be administered orally,
intramuscularly or intravenously.
 D2 antagonist exacerbate extrapyramidal symptoms so use with caution in
patients with Parkinsonism.
 Benzodiazepines (e.g. lorazepam) can cause, worsen or prolong delirium
(paradoxical disinhibitions or over-sedation) so do not use unless treating
delirium due to alcohol or benzodiazepine withdrawal.
 Avoid the use of restraints which may worsen agitation and cause injury.
 If restraints are necessary, use the least means appropriate for the situation and
remove them as soon as the patient meets criteria for release.

A 75-year-old lady is found lying on the floor and taken to hospital. She is drowsy,
disorientated in time and place, distractible and unable to give any history. She thinks
you are trying to kill her. She is febrile and hypotensive but has no neurological signs or
injuries. Blood tests and X-rays are performed. She is given oxygen and antibiotics for
the clinical suspicion of septicemia. Her agitation worsens but settles with lorazepam.
The GP tell you that there is no past history to note. Blood cultures grow an organism
sensitive to the antibiotic. Her condition improves over 72 hours. The lorazepam is tailed
off and her cognitive function returns to normal.
Diagnosis: acute confusional state secondary to bacteremia

39
MOOD DISORDES
 Broadly speaking the emotions can be described as 2 main types:
 Affect, which is a short-lived emotional response to an idea or an event. It
is the external expression of mood (it is objective).
 Mood: This is a description of one’s internal emotional state (it is
subjective).
 According to these definitions, depression and mania are mood disorders and
not ‘affective disorders’ as they have been called so frequently.
 Both external and internal stimuli can trigger moods, which may be labeled
as sad, happy, angry, irritable and so on.
 It is normal to have a wide range of moods and to have a sense of control
over one’s moods.
 Patients with mood disorders (also called affective disorders) experience an
abnormal range of moods and lose some level of control over them.
 Distress may be caused by the severity of their moods and the resulting
impairment in social and occupational functioning.
 Mood episodes are distinct periods of time in which some abnormal mood is
present. They include hypomanic episode, manic episode, and a depressive
episode.
 Mood disorders are defined by their patterns of mood episodes. They include
major depressive disorder, Bipolar disorder (bipolar I and II), persistent
depressive disorder (dysthymia) and cyclothymic disorder. Some may have
psychotic features (delusions or hallucinations).
 When patients have delusions and hallucinations due to underlying mood
disorders, they are usually mood congruent. For example, depression causes
psychotic themes of paranoia and worthlessness, and mania causes psychotic
themes of grandiosity and invincibility.

CLASSIFICATION
 According to the ICD-10, the mood disorders are classified as follows:
1. Manic episode
2. Depressive episode
3. Bipolar mood (affective) disorder
4. Recurrent depressive disorder
5. Persistent mood disorder (including cyclothymia and dysthymia)
6. Other mood disorder (including mixed affective episode and recurrent
brief depressive disorder)

40
MOOD EPISODES
MANIC EPISODE
 This is a distinct period of abnormally and persistently elevated, expansive
or irritable mood and increased goal-directed activity or energy, lasting at
least 1 week (or any duration if hospitalization is necessary).
 Episodes tend to last usually 3-4 months, followed by complete clinical
recovery. The future episodes can be manic, depressive or mixed.
 Manic episodes include at least 3 of the following (4 if mood is only
irritable):
 Distractibility
 Inflated self-esteem or grandiosity
 Increase in goal-directed activity (socially, at work, or sexually) or
psychomotor agitation
 Decreased need for sleep
 Flight of ideas or racing thoughts
 More talkative than usual or pressured speech (rapid and uninterruptible)
 Excessive involvement in pleasurable activities that have a high risk of
negative consequences (e.g. shopping sprees, sexual indiscretions)
 Symptoms are not attributable to the effects of a substance (drug or
medication) or another medical condition, and they must cause clinically
significant distress or social/occupational impairment.
 Greater than 50% of manic patients have psychotic symptoms.

SYMPTOMS OF MANIA: “DIG FAST”


D- Distractibility
I- Insomnia/ impulsive behavior
G- Grandiosity

F- Flight of ideas/racing thoughts


A- Activity/Agitation
S-Speech (pressured)
T- Thoughtlessness

 A manic episode is a psychiatric emergency, severely impaired judgment can


make a patient dangerous to self and others.

41
HYPOMANIC EPISODE
 A hypomanic episode is a distinct period of abnormally and persistently
elevated, expansive or irritable mood, and abnormally and persistently
increased goal-directed activity or energy, lasting at least 4 consecutive days
that includes at least 3 of the symptoms listed for the manic episode criteria
(4 if mood is only irritable)
MANIA HYPOMANIA
Lasts at least 7 days Lasts at least 4 days
Causes severe impairment in social or No marked impairment in social or
occupational functioning occupational functioning
May necessitate hospitalization to Does not require hospitalization
prevent harm to self or others
May have psychotic features No psychotic features

 Note: for mixed features, criteria are met for a manic or hypomanic episode
and at least 3 symptoms of a major depressive episode are present for the
majority of the time. These criteria must be present nearly every day for at
least 1 week.
MAJOR DEPRESSIVE EPISODE
 The typical depressive episode is characterized by the following features
(which should last for at least 2 weeks for a diagnosis to be made):
 Depressed mood most of the time
 Anhedonia (loss of interest in pleasurable activities)
 Change in appetite or weight (increase or decrease)
 Feelings of worthlessness or excessive guilt
 Insomnia or hypersomnia
 Diminished concentration
 Psychomotor agitation or retardation (i.e. restlessness or slowness)
 Fatigue or loss of energy
 Recurrent thoughts of death or suicide
 Symptoms are not attributable to the effects of a substance (drug or
medication) or another medical condition, and they must cause clinically
significant distress or social/occupational impairment.

42
SYMPTOMS OF DEPRESSION: “SIG E.
CAPS” (Prescribe Energy Capsules)
S- Sleep
I- Interest
G-Guilt

E- Energy
C- Concentration
A-Appetite
P- Psychomotor activity
S-Suicidal ideation

MOOD DISORDERS
 Mood disorders often have chronic courses that are marked by relapses with
relatively normal functioning between episodes.
 Like most psychiatric diagnoses, mood episodes may be caused by another
medical condition or drug (prescribed or illicit), therefore, always investigate
medical or substance-induced causes before making a primary psychiatric
diagnosis.
 Medical condition causing depressive episodes:
 Cerebrovascular disease (stroke, myocardial infraction)
 Endocrinopathies: diabetes mellitus, Cushings syndrome, Addison
disease, Hypoglycemia, Hyper/Hypothyroidism, Hyper/Hypocalcemia
 Parkinson’s disease
 Viral illnesses e.g. mononucleosis
 Carcinoid syndrome
 Cancer (especially lymphoma and pancreatic carcinoma)
 Collagen vascular disease e.g. Systemic lupus erythematosus
 Medical conditions causing manic episodes:
 Metabolic: hyperthyroidism
 Neurological disorders: temporal lobe seizures, multiple sclerosis
 Neoplasms
 HIV infection

43
 Substance/Medication-induced depressive disorder: EtOH,
antihypertensives, barbiturates, corticosteroids, levodopa, sedative-
hypnotics, anticonvulsants, antipsychotics, diuretics, sulfonamides,
withdrawal from stimulants e.g. cocaine, amphetamines
 Substance/ Medication-induced Bipolar disorder: Antidepressants,
sympathomimetics, dopamine, corticosteroids, levodopa, bronchodilators,
cocaine, amphetamines
DEPRESSIVE DISORDER
 Depression is a mood disorder that causes persistent feeling of sadness and
loss of interest.
 It is called major depressive disorder or clinical depression.
 Depressive disorder is common. The risk is greatest in women and in those
with a positive family history.
 Major depressive disorder is marked by episodes of depressed mood
associated with loss of interest in daily activities.
 Patients may not acknowledge their depressed mood or may express vague
somatic complaints (fatigue, headache, abdominal pain, muscle tension etc.)
 Onset can happen at any age but the age of onset peaks in the 20s.
 Depression can increase mortality for patients with other comorbidities such
as diabetes, stroke and cardiovascular disease.

44
CLINICAL FEATURES AND DIAGNOSIS
 The term ‘depression’ is widely used to describe low mood but in order for a
diagnosis of depressive disorder to be made there are a number of key
features that must be present for at least 2 weeks.
ICD-10 CRITERIA FOR DEPRESSIVE DISORDER
A B
 Persistent low mood  Reduced concentration and
 Loss of interest or pleasure attention
(anhedonia)  Reduced self-esteem and self-
 Fatigue or low energy (anergia) confidence
 Ideas of guilt and worthlessness
 Hopelessness about the future
 Suicidal thoughts
 Disturbed sleep
 Diminished appetite
The severity of the episode is determined by how many of these features are
present:
 Mild: 2 or more from A + 2 from B (has some impact on daily life)
 Moderate: 2 or more from A + 3 from B (has significant impact on
daily life)
 Severe: all 3 from A + 4 or more from B (impossible to get through
daily life)
If there has been more than one discrete episode this is termed recurrent
depressive disorder.

 Depressive disorder is also associated with a number of other symptoms that


relate to these core diagnostic features.
 These include psychomotor retardation (slowed movements and thinking) or
agitation, loss of libido, constipation and amenorrhea.
 This group of symptoms are sometimes referred to as somatic syndrome.
 Psychological symptoms of depression:
 Continuous low mood or sadness, feeling hopeless and helpless, having
low self-esteem, feeling tearful, feeling guilt-ridden, feeling irritable and
intolerant of others.
 Having no motivation or interest in things, finding it difficult to make
decisions, not getting any enjoyment out of life, feeling anxious or
worried, having suicidal thoughts or thoughts of harming one’s self.

45
 Physical symptoms:
 Moving slowly (psychomotor retardation), speaking more slowly than
usual, changes in appetite or weight (usually decreased, but sometimes
increased), constipation, unexplained aches and pains
 Lack of energy, low sex drive (loss of libido), changes to menstrual cycle,
disturbed sleep (finding it difficult to fall asleep at night or waking up
very early in the morning or excessive sleepiness)
 Social symptoms:
 Not doing well at work, avoiding contact with friends, taking part in
fewer social activities, neglecting one’s hobbies and interests, having
difficulties in one’s home and family life.
RISK FACTORS
 Biochemistry: differences in certain chemicals (neurotransmitters) in the
brain may contribute to symptoms of depression.
 Genetics: depression can run in families. For example, if one identical twin
has depression, the other has a 70% chance of having the illness sometime in
life.
 Personality: people with low self-esteem, who are easily overwhelmed by
stress, or who are generally pessimistic appear to be more likely to
experience depression.
 Environmental factors: continuous exposure to violence, neglect, abuse or
poverty may make some people more vulnerable to depression.
PATHOPHYSIOLOGY
 The precise cause of depression is unknown, but major depressive disorder is
believed to be a heterogeneous disease with biological, genetic,
environmental and psychosocial factors contributing.
 The monoamine theory of depression postulates that depression results from
a central deficiency in the monoamine neurotransmitters serotonin (5-HT),
norepinephrine and dopamine.
 Evidence for this includes: antidepressants exert their therapeutic effect
by increasing catecholamines, decreasing cerebrospinal fluid levels of 5-
hydroxyindolacetic acid (5-HIAA), the main metabolite of serotonin in
depressed patients with impulsive and suicidal behavior.
 Increased sensitivity of beta-adrenergic receptors in the brain has also
been postulated in the pathogenesis of major depressive disorder.
 Other reported physiological features include increased cortisol and a
blunted TSH response. GABA, Glutamate and endogenous opiates may
additionally have a role.

46
 However, there is no widely accepted and definitively proven biological
model of depression.
 Psychosocial/life events: multiple adverse childhood experiences are a risk
factor for later developing major depressive disorder.
 Loss of a parent before age 11 is associated with the later development of
major depression.
 Genetics: first degree relative are 2 to 4 times more likely to have major
depressive disorder. Concordance rate for monozygotic twins is <40% and
10-20% for dizygotic twins.

OTHER TYPES OF DEPRESSION


 Postnatal depression
 Some women develop depression after they have a baby.
 It is treated in a similar way to other types of depression.
 Common methods include talk therapy and antidepressant medicines
 Bipolar disorder
 Also known as manic depression
 In bipolar disorder there are spells of both depression and excessively
high mood (mania)
 The depression symptoms are similar to clinical depression but the bouts
of mania can include harmful behavior such as gambling, going on
spending sprees and having unsafe sex etc.
 Seasonal affective disorder (SAD)
 It is also known as “winter depression”
 It is the type of depression with a seasonal pattern usually related to
winter.
 Dysthymia
 This term is used to describe longstanding mild depressive symptoms.
Dysthymia is often associated with other psychiatric or physical illness
and can also co-occur with depression- a condition sometimes termed
‘double depression’.
 Psychotic depression
 Depression at its most severe becomes psychotic depression (depressive
psychosis)
 Worries and perceive misdemeanours become delusional in intensity.
 The patient may believe that they, or a part of them is dead (Cotard’s
syndrome), they may experience auditory hallucinations, which are often
derogatory in nature. Suicide risk is high.

47
 Psychomotor retardation can increase to the point where the person sits
motionless and mute-depressive stupor. This often used to be fatal (from
dehydration), it now calls for emergency electroconvulsive therapy
(ECT).
 Psychotic depression must be distinguished from other psychoses. This is
based on the presence of other depressive symptoms and the mood
congruity of the delusions and hallucinations.
 Atypical depression
 For some individuals depression is associated with increased sleep,
increased appetite and phobic anxiety.
 This is often termed atypical depression and tends to respond better to
monoamine oxidase inhibitors (MAOIs) rather than selective serotonin
reuptake inhibitors (SSRIs).
 Reactive and endogenous depression
 This dated classification divided depression into ‘reactive depression’,
brought on by a stressful life event and ‘endogenous depression’
supposedly occurring from within the patient, with no clear external
cause.
 Endogenous depression was thought to be more heritable and more
responsive to antidepressant treatment. Research into depression has
shown that such a division does not exist and these terms are rarely used
now.
 Mixed anxiety and depressive disorder
 Anxiety symptoms are common in depressive disorder and when
symptoms of both disorders are present but not individually sufficient
enough to meet criteria for a diagnosis of a mood disorder or an anxiety
disorder this is described as a mixed anxiety and depressive disorder.
TREATMENT
 Non-pharmacological: Psychotherapy
 Psychotherapy or “talk-therapy” is sometimes used alone for treatment of
mild depression.
 For moderate to severe depression, psychotherapy is often used along
with antidepressant medications.
 Cognitive behavioral therapy (CBT) has been found to be effective in
treating depression.
o CBT is a form of therapy focused on the present and problem solving.
o CBT helps a person to recognize distorted thinking and then change
behaviors and thinking.

48
 Psychotherapy may involve only the individual but it can include others.
For example, family or couples therapy can help address issues within
these close relationships.
 Group therapy involves people with similar illnesses.
 Pharmacological: Antidepressants (SSRIs)
 Produce some improvement within the first week or two of use
 Full benefits may not be seen for 2 or 3 months.
 If a patient feels little or no improvement after several weeks, the doctor
can alter the dose of the medication or add or substitute another
antidepressant.
 In some situations, other psychotropic medication may be helpful.
 Electroconvulsive therapy (ECT)
 This is a medical treatment most commonly used for patients with severe
major depression or bipolar disorder who have not responded to other
treatments.
 It involves a brief electrical stimulation of the brain while the patient is
under anesthesia.
 A patient typically receives ECT 2 to 3 times a week for a total of 6 to 12
treatments.
 ECT has been used since 1940s, and many years of research have led to
major improvements.
 It is usually managed by a team of trained medical professionals
including a psychiatrist, an anesthesiologist and a nurse/clinical
officer/medical licentiate.

49
BIPOLAR DISORDER
 This disorder, earlier known as manic depressive psychosis is characterized
by recurrent episodes of mania/hypomania (elated mood) and depression in
the same patient at different times.
 The elevated mood is significant and is known as mania or hypomania,
depending on its’s severity, or whether symptoms of psychosis are
present.
 These episodes can occur in any sequence.
 The presence of elated mood alone is sufficient for the diagnosis to be made.
 The patients with recurrent episodes of mania (unipolar mania) are also
classified here as they are rare and often resemble the bipolar patients in
their clinical features.
 Bipolar mood disorder has an early age of onset of third decade. Unipolar
depression, on other hand is common in 2 ages groups: late third decade and
fifth to sixth decades.
 Unipolar depression usually last longer than bipolar depression.
 An average manic episode lasts for 3-4 months while a depressive episode
lasts from 4-6 months.
 With rapid institution of treatment, the major symptoms of mania are
controlled within 2 weeks and of depression within 6-8 weeks.
 The current episode in bipolar disorder is specified as one of the following
(ICD-10):
 Hypomanic
 Manic without psychotic symptoms
 Manic with psychotic symptoms
 Mild or moderate depression
 Severe depression, without psychotic symptoms
 Severe depression, with psychotic symptoms
 Mixed or
 In remission

50
 Bipolar mood disorder is further classified into:
 Bipolar I: characterized by episodes of severe mania and severe
depression
 Bipolar II: characterized by episodes of hypomania (not requiring
hospitalization) and severe depression
BIPOLAR I DISORDER
 Bipolar I disorder involves episodes of mania and of major depression
however episodes of major depression are not required for the diagnosis.
 It is also known as manic-depression.
 The only requirement for this diagnosis is the occurrence of a manic episode
(5% of patients experience only manic episodes). Between manic episodes,
there may be interspersed euthymia, major depressive episodes, or
hypomanic episodes but none of these are required for the diagnosis.
 Bipolar I disorders may have psychotic features (delusions or hallucinations)
these can occur during major depressive or manic episodes.
 Remember to always include bipolar disorder in the differential diagnoses
of a psychotic patient.
 Rapid cycling is defined by the occurrence of 4 or more mood episodes in 1
year (major depressive, hypomanic or manic).
 Women and men are equally affected.
 No ethnic differences are seen.
 Onset is usually before age 30, mean age of first mood episode is 18.
ETIOLOGY
 Biological, environmental, psychosocial and genetic factors are all
important.
 First degree relatives of patients with bipolar disorder are 10 times more
likely to develop the illness.
 Concordance rates for monozygotic twins are 40-70%, and rates for
dizygotic twins range from 5 to 25%.
 Bipolar I has the highest genetic link of all major psychiatric disorders.

51
TREATMENT
 Pharmacotherapy:
 Lithium is a mood stabilizer, 50-70% treated with lithium show partial
reduction of mania. Long-term use reduces suicide risk. Acute overdose
can be fatal due to its low therapeutic index.
o Side-effects: weight gain, tremor, gastrointestinal disturbances,
fatigue, cardiac arrhythmias, seizures, goiter/hypothyroidism,
leukocytosis (benign), coma (in toxic doses), polyuria (nephrogenic
diabetes insipidus), polydipsia, alopecia, metallic taste.
 The anticonvulsants carbamazepine and valproic acid are also mood
stabilizers. They are particularly useful for rapid cycling bipolar disorder
and those with mixed features.
 Atypical antipsychotics (risperidone, olanzapine, quetiapine, ziprasidone)
are effective as both monotherapy and adjunct therapy for acute mania. In
fact, many patients (especially with severe mania and/or with psychotic
features) are treated with a combination of a mood stabilizer and
antipsychotic, studies have shown a greater and faster response with
combination therapy.
 Antidepressants are discouraged as monotherapy due to concerns of
activating mania or hypomania. They are occasionally used to treat
depressive episodes when patients concurrently take mood stabilizers.
 Treatment for bipolar disorder includes lithium, valproic acid and
carbamazepine (for rapid cyclers), or second-generation antipsychotics.
Lithium remains the gold standard, particularly due to demonstrated
reduction in suicide risk.
 Psychotherapy:
 Supportive psychotherapy
 Family therapy
 Group therapy (may prolong remission once the acute manic episode has
been controlled)
 Electroconvulsive therapy:
 Works well in treatment of manic episodes.
 Some patients require more treatments (up to 20) than for depression.
 Especially effective for refractory or life-threatening acute mania or
depression.
 ECT is the best treatment for a pregnant woman who is having a manic
episode. It provides a good alternative to antipsychotics and can be used
with relative safety in all trimesters.

52
BIPOLAR II DISORDER
 Alternatively called recurrent major depressive episodes with hypomania.
 History of one or more major depressive episodes and at least one
hypomanic episode is needed for diagnosis. If there has been a full manic
episode, even in the past, then the diagnosis is bipolar I, not bipolar II
disorder.
 Bipolar II is frequently misdiagnosed as unipolar depression and thereby
inappropriately treated.
 Prevalence is unclear, with some studies showing that it occurs more/less
than bipolar I.
 It may be slightly more common in women.
 A patient with a history of postpartum mania has a high risk of relapse
with future deliveries and should be treated with mood stabilizing agents
as prophylaxis. However, some of these medications may be
contraindicated in breast-feeding.
 Onset usually before age 30.
 No ethnic differences seen.
 Etiology is the same as bipolar I.
 Bipolar II tends to be chronic, requiring long-term treatment though it carries
a better prognosis than bipolar I.
 Treatment is similar to bipolar I.

53
PERSISTENT DEPRESSIVE DISORDER (DYSTHYMIA)
 Patients with persistent depressive disorder (dysthymia) have chronic
depression most of the time, and they have discrete major depressive
episodes.
 It is more common in women and has an onset often in childhood,
adolescence and early adulthood.
 Dysthymia is characterized by:
 Depressed mood for the majority of the time most days for at least 2 years
(in children or adolescents for at least 1 year)
 At least 2 of the following:
o Poor concentration or difficult making decisions
o Feelings of hopelessness
o Poor appetite or overeating
o Insomnia or hypersomnia
o Low energy or fatigue
o Low self-esteem
 During the 2-year period
o The person has not been without the above symptoms for >2 months
at a time
o May have major depressive episode(s) or meet criteria for major
depression continuously.
o The patient must never have had a manic or hypomanic episode (this
would make the diagnosis bipolar disorder or cyclothymic disorder
respectively).
 Treatment:
 Combination treatment with psychotherapy and pharmacotherapy is more
efficacious than either alone.
 Cognitive therapy, interpersonal therapy, and insight-oriented
psychotherapy are the most effective.
 Antidepressants found to be beneficial include SSRIs, TCAs and MAOIs.
CYCLOTHYMIC DISODER
 This is characterized by alternating periods of hypomania and periods with
mild-to-moderate depressive symptoms.
 It may coexist with borderline personality disorder.
 It occurs equally in males and females with an onset usually age 15-25.
 Cyclothymic disorder is characterized by:
 Numerous period with hypomanic symptoms (but not a full hypomanic
episode) and periods with depressive symptoms (but not full major
depressive episode) for at least 2 years.

54
 The person must never have been symptom free for >2 months during
those 2 years.
 No history of major depressive episode, hypomania or manic episode.
 Treatment is with antimanic agents (mood stabilizers or second-generation
antipsychotics) as used to treat bipolar disorder.

55
NEUROTIC, STRESS-RELATED AND
SOMATOFORM DISORDERS
 The terms neurosis and psychosis are currently not widely used. The
definitions and description of these terms are far from perfect and there are
clear exceptions to the rules.
 The ICD-10 still mentions the term neurotic in the classification, it
discourages the use of the terms neurosis and psychosis.
 In ICD-10, ‘neurotic, stress-related and somatoform disorders’ have been
classified into the following types:
 Phobic anxiety disorder: simple phobia/specific phobias, agoraphobia
 Other anxiety disorders (simply referred to as Anxiety disorders here):
generalized anxiety disorder
 Obsessive compulsive disorder
 Neurosis (‘to do with nerves’) or neurotic disorder are the traditional terms
for those symptoms when they are not due to an organ brain disease,
psychosis or personality disorder.
 The symptoms of neurosis are emotional, cognitive, behavioral and somatic.
 Neuroses are often but not always associated with an external stressor.
 Anxiety is the primary emotion in all these disorders, although depressed
mood is often present.
 The cognitions are worries, fears and concerns that are inappropriate or
excessive but (by definition) not delusional.
 Behaviors include avoidance and other strategies intended to reduce
anxiety such as repeated checking.
 Somatic (physical) symptoms, not explained by a medical disease but
associated with tension, autonomic arousal and hyperventilation (such as
aches and pains, bowel disturbance and breathlessness), are common.
 Neurotic disorders may present with the emotion of anxiety (e.g. attacks of
panic) but may also present with one of the other symptoms such as
cognitions e.g. recurrent worries about having a serious medical condition,
behavior e.g. the agoraphobic unable to leave her house or somatic
symptoms e.g. palpitations.

56
Figure 3: Classification of neurotic, stress-related and somatoform disorders

57
ANXIETY DISORDER
 Anxiety is the commonest psychiatric symptom in clinical practice and
anxiety disorders are one of the commonest psychiatric disorders in general
population.
 Anxiety is a ‘normal’ phenomenon, which is characterized by a state of
apprehension or unease arising out of anticipation of danger.
 Anxiety is an individual’s emotional and physical fear response to a perceived
threat.
 Anxiety is often differentiated from fear, as fear is an apprehension in response
to an external danger while in anxiety the danger is largely unknown (or
internal).
 Normal anxiety becomes pathological when it causes significant subjective
distress and/or impairment in functioning of an individual.
 Pathologic anxiety occurs when symptoms are excessive, irrational, out of
proportion to the trigger or are without an identifiable trigger.
 Anxiety disorders are caused by a combination of genetic, biological,
environment and psychosocial factors.
 Major neurotransmitter system implicated: norepinephrine (NE), serotonin (5-
HT), and gamma-aminobutyric acid (GABA).
 Anxiety disorders affect more women compared to men (2:1).
 Symptoms of anxiety:
 Physical symptoms:
o Constitutional: fatigue, diaphoresis (sweating), shivering (trembling)
o Autonomic and visceral symptoms:
 Neurologic/musculoskeletal: dizziness, lightheadedness,
paresthesias, tremors, insomnia, muscle tension, agitation,
sweating, flushes, mydriasis
 Cardiac: Chest pain, palpitations, tachycardia, hypertension.
 Pulmonary: hyperventilation, shortness of breathing, dyspnea,
 Gastrointestinal: abdominal discomfort, dry mouth, anorexia,
nausea, emesis, diarrhea, constipation
 Genitourinary: frequency and hesitancy of micturition
o Motor symptoms: tremors, restlessness, muscle twitches, fearful facial
expression
 Psychological symptoms
o Cognitive symptoms: poor concentration, distractibility, hyperarousal,
vigilance or scanning, negative automatic thoughts.
o Perceptual symptoms: derealization, depersonalization

58
o Affective symptoms: diffuse, unpleasant and vague sense of
apprehension, fearfulness, inability to relax, irritability, fear of loss of
control, feeling of impending doom/dread (when severe)
o Other symptoms: insomnia (initial), increased sensitivity to noise,
exaggerated startle response obsessions and compulsions.
 Risk factors for most anxiety disorders:
 Family history (mild).
 Women are at higher risk, except OCD and social anxiety disorders.
 Onset is usually in teens and 20s. However, may rumble on untreated for
years. Exception is GAD which is commonest in 40s and 50s.
 Life stressors: this can include physical illness.
 Medications and substances that cause anxiety: alcohol, sedatives/hypontics,
cannabis, hallucinogens (phencyclidine, lysergic acid,
methylenedioxymethamphetamine), stimulants (amphetamines, cocaine),
caffeine, tobacco, and opioids.

 Anxiety disorders are divided into 3 main subtypes:


 Paroxysmal (panic) disorder
 Phobic anxiety
 Generalized anxiety disorder

59
 Treatment includes pharmacological drugs and psychotherapy.
 Pharmacological drugs: drugs are used to achieve symptomatic relief and
continue treatment for at least 6 months before attempting to titrate off
medications.
o First-line: Selective serotonin reuptake inhibitors (SSRIs) e.g. sertraline
and serotonin-norepinephrine reuptake inhibitors (SNRIs) e.g.
venlafaxine. Note: SSRIs typically take about 4-6 weeks to become
fully effective and higher doses (than used in treating depression) are
generally required.
 Sertraline initially 25mg daily for 1 week then increased to 50mg
daily, then increased in steps of 50mg at intervals of at least 1
week if required, maximum dose is 200mg per day. Increase dose
only if response is partial and if drug is tolerated.
o Benzodiazepine work quickly and effectively, but they all can be
addictive. Minimize the use, duration and dose. Benzodiazepines
should be avoided in patients with a history of substance use disorders,
particularly alcohol.
 Note: if a patient has a comorbid depressive disorder, consider
alternative to benzodiazepines as they may worsen depression.
Use benzodiazepines to temporarily bridge patients until long-
term medication becomes effective.
 Benzodiazepines should be avoided except for short-term relief
during crises.
o Busiprone (5-HT1a partial agonist): this is a non-benzodiazepine
anxiolytic, however it is not commonly used due to mininmal efficacy
and often only prescribed as augmentation.
o Beta blockers (e.g. propranolol): may be used to help control autonomic
symptoms with panic attack or performance anxiety.
o Tricyclic antidepressants (TCAs) and monoamine oxidase inhibitors
(MAOIs): May be considered if first-line agents are not effective. Their
side-effect profile makes them less tolerable.
o Note: warn that drugs will take 1 week to take effect and may initially
cause transient increase in anxiety or agitation. Discuss suicide risk if
under 30, offer proton pump inhibitors if on NSAIDs or aspirin, and
warn against sudden cessation due to withdrawal effects.
 Psychotherapy:
o Cognitive behavioral therapy (CBT): has been proven effective for
anxiety disorders. CBT examines the relationship between anxiety-
driven cognitions (thoughts), emotions and behaviors.

60
o Psychodynamic psychotherapy: facilitates understanding and insight
into the development of anxiety and ultimately increases anxiety
tolerance.

PANIC ATTACKS
 Panic attacks are a type of fear response involving an abrupt surge of intense
anxiety which may be triggered or occur spontaneously.
 Panic attacks peak within minutes (10-20 mins) and usually resolve within
half an hour.
 Patients may continue to feel anxious for hours afterwards and confuse this
for a prolonged panic attack.
 Although classically associated with panic disorder, panic attacks can also be
experienced with other anxiety disorders, psychiatric disorders and other
medical conditions.
 Symptoms of panic attacks “Da PANICS” (4 symptoms have to be present
for a panic attack to be diagnosed)
 D- dizziness, disconnectedness, derealization (unreality),
depersonalization (detached from self)
 P- palpitations, paresthesias
 A- abdominal distress
 N- numbness, nausea
 I- Intense fear of dying, losing control or “going crazy”, intense
apprehension
 C- chills, chest pain
 S- sweating, shaking, shortness of breath
 When a patient presents with a panic attack, rule out potentially life-
threatening medical conditions such as heart attack, thyrotoxicosis and
thromboembolism.
 Panic attacks may co-exisit with other psychiatric conditions such as
depressive disorder, post-traumatic stress disorder and substance abuse.

PANIC DISORDER
 Panic disorder is characterized by spontaneous, recurrent panic attacks (2 or
more). These attacks occur suddenly, “out of the blue”.
 Patients may also experience panic attacks with a clear trigger.
 The frequency of attacks ranges from multiple times per day to a few monthly.

61
 Patients develop debilitating anticipatory anxiety about having future attacks-
“fear of the fear” this can lead to avoidance behaviors and become so severe
as to leave patients homebound (i.e. agoraphobia).
 Panic disorder associated with somatic symptoms and unpleasant feelings of
depersonalization or derealization.
 Individuals with panic disorder may interpret their symptoms as a physical
health problem.
 In panic disorder, the attacks are recurrent over a period of at least 1 month.
Panic disorder is characterized by:
 Recurrent, unexpected panic attacks without an identifiable trigger.
 One or more of panic attacks followed by more than 1 month of continuous
worry about experiencing subsequent attacks or their consequences and/or
a maladaptive change in behaviors e.g. avoidance of possible triggers.
 Not caused by the direct effects of a substance, another mental disorder, or
another medical condition.
 Panic disorder has a greater risk if first-degree relative is affected. It is
associated with increased incidence of stressors (especially loss) prior to onset
of disorder, history of childhood physical or sexual abuse.
 Panic disorder is higher in women compared to men (2:1).
 Median age of onset: 20-24 years old.
 Other comorbid syndromes include other anxiety disorders (especially
agoraphobia), bipolar disorder, depression and alcohol use disorder.
 Treatment:
 Pharmacotherapy:
o First-line: SSRIs (e.g. sertraline, citalopram, escitalopram). Allow 12
weeks to see if effective, they have sedative and relaxing effects.
Counsel as always on side effects.
 Sertraline initially 25mg daily for 1 week then increased to 50mg
daily, then increased in steps of 50mg at intervals of at least 1
week if required, maximum 200mg per day. Increased only if
response is partial and if drug is tolerated.
o Can switch to TCAs (clomipramine, imipramine) if SSRIs not effective.
o Continue for at least 6 months if effective.
o Can use benzodiazepines (clonazepam, lorazepam) as scheduled or
PRN (as needed) especially until the other medications reach full
efficacy.
o If severe antiseizure medication can be prescribed.
 Psychotherapy:
o Cognitive behavioral therapy- 7-14 weekly sessions. It involves:

62
 Patient learning about the panic disorder and how to identify
certain symptoms.
 Monitor panic attacks using a diary
 Breathing and relaxation techniques
 Change in beliefs about panic attacks
 The patient allowing themselves to be exposed to certain
situations that provoke panic attacks
o Psychoeducation for patient and family.
o Social: Peer support groups
PHOBIC DISORDER
 Phobia is defined as an irrational fear of a specific object, situation or activity,
often leading to persistent avoidance of the feared object, situation or activity.
 The common types of phobias are:
 Agoraphobia
 Social phobia
 Specific (Simple) phobia
 Characteristics of phobia:
 Presence of the fear of an object, situation or activity
 The fear is out of proportion to the danger perceived
 Patient recognizes the fear as irrational and unjustified (insight is present)
 Patient is unable to control the fear and is very distressed by it
 This leads to persistent avoidance of the particular object, situation or
activity
 Gradually, the phobia and the phobic object become a preoccupation with
the patient, resulting in marked distress and restriction of the freedom of
mobility (afraid to encounter the phobic object, phobic avoidance).
AGORAPHOBIA
 Agoraphobia is intense fear of being in public places where escape or
obtaining help may be difficult.
 It often develops with panic disorder.
 The course of the disorder is usually chronic.
 Avoidance behaviors may become as extreme as complete confinement to the
home.
 Onset is usually before age 35.
 In Agoraphobia:
 Intense fear/anxiety about >2 situations due to concern of difficult escaping
or obtaining help in case of panic or other humiliating symptoms:
o Outside of the home alone

63
o Open spaces (e.g. bridges)
o Enclosed places (e.g. stores)
o Public transportation (e.g. trains)
o Crowds/lines
 The triggering situations cause fear/anxiety out of proportion to the
potential danger posed, leading to endurance of intense anxiety, avoidance
or requiring a companion. This holds true even if the patient suffers from
a medical condition such as inflammatory bowel disease (IBS) which may
lead to embarrassing public scenarios.
 Symptoms cause significant social or occupational dysfunction.
 Symptoms last more 6 months.
 Symptoms not better explained by another mental disorder.
 Comorbid diagnoses include other anxiety disorders, depressive disorders and
substance use disorders.
 During diagnosis a history is taken asking specific questions e.g. medications
taken as well as a physical examination to rule out conditions that may present
with anxiety.
 Treatment is essentially the same as panic disorder. (CBT and SSRIs)
 Psychotherapy: CBT
o Systematic desensitization: a person is gradually exposed to the feared
object/situation/activity
 Patient identifies the anxiety
 Patient learns coping techniques
 The patient uses learnt techniques to cope with various situations
 Drugs: SSRIs
o Sertraline initially 25mg daily for 1 week then increased to 50 mg daily,
then increased in steps of 50mg at intervals of at least 1 week if required,
maximum dose is 200mg per day. Increase only if response is partial
and if drug is tolerated.

SOCIAL PHOBIA
 Fear of social situations and interactions.
 Fearful of being embarrassed or judged by others.
 Anticipatory anxiety may be present
 Physical symptoms: trembling, blushing and derealization. Person may get
performance anxiety (a specific type of social anxiety restricted to
performance).

64
 In social phobia:
 Individuals fear acting in a way that might make them get judged.
 The phobia interferes with normal routine and relationships.
 The fear of anxiety is persistent (>6 months)
 Treatment same as agoraphobia.
 Beta-blockers may be used for performance anxiety.
SPECIFIC (SIMPLE) PHOBIA
 These are irrational fears of specific objects or situations e.g.
 Fear of animals e.g. arachnophobia (fear of spiders), alektorophobia (fear
of chickens), ophidiophobia (fear of snakes), cynophobia (fear of dogs)
 Fear of natural environment e.g. nyctophobia (Fear of the dark),
Astrophobia (fear of lightening)
 Fear of blood and needles (hemophobobia)
 Fear of situations e.g. flying (aviophobia), claustrophobia (fear of enclosed
spaces), Acrophobia (fear of heights)
 Others e.g. fear of clowns (coulrophobia)
 Treatment same as agoraphobia.
 Treat any co-morbidity.
GENERALIZED ANXIETY DISORDER
 Patients with GAD have persistent, excessive anxiety about many aspects of
their daily lives (e.g. money, work, finances, home, relationships)
 Some symptoms GAD include irritability, edginess, impaired concentration,
restlessness and “blacking out” of the mind.
 Often, they experience somatic symptoms including insomnia, fatigue, muscle
tension/aches (clench jaw or teeth grinding) and bowel symptoms (diarrhea or
constripation).
 Generalized anxiety disorder (GAD) is characterized by an insidious onset in
the 3rd decade and a stable, usually chronic course which may or may not be
punctuated by repeated panic attacks (episodes of acute anxiety).
 GAD is persistent anxiety associated with chronic uncontrollable and
excessive worry. It may fluctuate in severity but is not paroxysmal (as with
panic), situational (as with phobia), life-long (as with personality disorder) or
clearly stress related (as with a stress-related disorder).
 GAD rates are higher in women compared to men (2:1).
 Age of onset: 30 years.
 The symptoms of anxiety should last for at least a period of 6 months for a
diagnosis of generalized anxiety disorder to be made and 3 of the mentioned
symptoms should be present (in children only 1 is needed).

65
 As anxiety is a cardinal feature of almost all psychiatric disorders, it is very
important to exclude other diagnoses.
 The most important differential diagnosis is from depressive disorders and
organic anxiety disorder.
 Comorbid diagnoses include other anxiety disorders and depressive disorders.
 Treatment:
 The most effective treatment approach combines psychotherapy and
pharmacology
 Cognitive behavioral therapy: teaches different ways of thinking, behaving
and reacting.
 Selective serotonin reuptake inhibitors (e.g. sertraline, citalopram) or SNRI
(e.g. venlafaxime)
 Can also consider a short-term course of benzodiazepines or augmentation
with buspirone
 Much less commonly used medications are TCAs and MAOIs

POST-TRAUMATIC STRESS DISORDER


 This is an anxiety disorder that develops after experiencing a traumatic event
where life or safety have been threatened e.g. war, natural disasters, car
crash, and rape.
 In PTSD the Fight, flight or fright response is dysfunctional.
CLINCIAL FEATURES
 Symptoms present within 6 months of event but can be later.
 Symptoms:
 Intrusive memories: experiencing trauma through thoughts and memories.
These may be triggered by thoughts or other external factors that remind
the individual about the event.
o Re-living flashbacks, dreams or vivid memories of the event
accompanied by distress
 Avoidance of reminders of the event e.g. certain people or places
 Hyperarousal/hypervigilance: autonomic arousal leading to poor sleep,
irritability, angry outbursts, extreme startle response and poor
concentration
 Negative thinking and mood. Dull/numbed emotions: feeling detached
from others.

66
MANAGEMENT
 Psychotherapy: Cognitive behavioral therapy
 Exposure therapy: exposure to fears in a safe and controlled environment
through mental imagery, writing or visits to the place where traumatic
event happened.
 Cognitive restructuring: patients are helped to make sense of their bad
memories.
 Trauma focused CBT (8-12 sessions): include psychoeducation, anxiety and
anger management, breathing techniques and exposure to triggers in a
controlled environment.
 Eye movement desensitization and reprocessing (EMDR): eye movements
while focusing on the memory.
 Pharmacological therapy:
 SSRIs: Sertraline (Zoloft) initially 50mg daily, then increased in steps of
50mg at intervals of at least 1 week if required, maintenance 50mg daily,
maximum 200mg per day.
 Alternatively, paroxetine (paxil) can also be used.
 Benzodiazepines can also be prescribed though their use is limited due to
dependence.
 Sleep-aids, Group therapy and Support groups have also been shown to be
effective.
 Treat PTSD before treating secondary co-morbid conditions unless they are
so severe that they prevent effective PTSD treatment.
 PTSD is most commonly associated with substance abuse (alcohol)

ACUTE STRESS DISORDER/ACUTE STRESS REACTION


 PTSD-like symptoms within 1 month following extreme stress e.g. car
accident, assault with onset typically in minutes to days.
 Symptoms include- dissociation and like PTSD- hypervigilance, avoidance,
intrusive thoughts and low mood.
 Management: trauma-focused CBT or active monitoring.

67
OBSESSIVE-COMPULSIVE DISORDER
 An obsession is defined as:
 An idea, impulse or image which intrudes into the conscious awareness
repeatedly.
 It is recognized as one’s own idea, impulse or image but is perceived as
ego-alien (foreign to one’s personality).
 It is recognized as irrational and absurd (insight is present).
 Patient tries to resist against it but is unable to.
 Failure to resist, leads to marked distress.
 An obsession is usually associated with compulsion(s). A compulsion is
defined as:
 A form of behavior which usually follows obsessions.
 It is aimed at either preventing or neutralizing the distress or fear arising
out of obsession.
 The behavior is not realistic and is either irrational or excessive.
 Insight is present, so the patient realizes the irrationality of compulsion.
 The behavior is performed with a sense of subjective compulsion (urge or
impulse to act).
 Lifetime prevalence: 2-3%.
 Mean age of onset: 20 years old.
 No gender difference in prevalence overall.
CLINICAL SYNDROMES
 ICD-10 classifies OCD into 3 clinical subtypes:
 Predominantly obsessive thoughts or ruminations
 Predominantly compulsive acts (compulsive rituals)
 Mixed obsessional thoughts and acts
 Depression is very commonly associated with obsessive compulsive disorder.
 4 clinical syndromes have been described in literature, although admixtures
are commoner than pure syndromes.
 Washers
o This is the commonest type. Here the obsession is of contamination with
dirt, germs, body excretions and the like.
o The compulsion is washing of hands or the whole body, repeatedly
many times a day.

 Checkers
o Persons constantly obsesses usually about the safety of something and
this leads to repetitive checking.

68
 Pure obsessions:
o This syndrome is characterized by repetitive intrusive thoughts,
impulses or images which are not associated with compulsive acts.
o The content is usually sexual or aggressive in nature.
o The distress associated with these obsessions is dealt usually by
counter-thoughts (such as counting) and not by behavioral rituals.
o A variant is obsessive rumination, which is a preoccupation with
thoughts.
o Here, the person repetitively ruminates in his mind about the pros and
cons of the thought concerned.
 Primary obsessive slowness:
o A relatively rare syndrome, it is characterized by severe obsessive ideas
and/or extensive compulsive rituals, in the relative absence of
manifested anxiety.
o This leads to marked slowness in daily activities.

TREATMENT
 Psychotherapy:
 Psychoanalytic psychotherapy is used in certain selected patients who are
psychologically oriented.
 Supportive psychotherapy is an important adjunct to other modes of
treatment. Supportive psychotherapy is also needed by the family
members.
 Behavior therapy and Cognitive behavior therapy:
 Behavior modification is an effective mode of therapy, with a success rate
as high as 80%, especially for the compulsive acts.
 The techniques used are listed below:
o Thought-stopping (and its modifications)
o Response prevention
o Exposure and response therapy
o Systemic desensitization
o Modelling
 Pharmacology:
 Antidepressants: some patients may improve dramatically with specific
serotonin reuptake inhibitors (SSRIs).
 Antipsychotics: these are occasionally used in low doses (e.g. haloperidol,
risperidone, olanzapine, aripiprazole, pimozide) in the treatment of severe,
disabling anxiety.

69
 Benzodiazepine (e.g. alprazolam, clonazepam) have a limited role in
controlling anxiety as adjuncts and should be used very sparingly.
 Buspirone has also been used beneficially as an adjunct for augmentation
of SSRIs, in some patients.
 Electroconvulsive therapy: in presence of severe depression with OCD, ECT
may be needed. ECT is particularly indicated when there is a risk of suicide
and/or when there is a poor response to the other modes of treatment.
However, ECT is not the treatment of first choice in OCD.
 Psychosurgery: psychosurgery can be used in treatment of OCD that has
become intractable and is not responding to other methods of treatment. It is
worth

70
PERSONALITY DISORDERS
 Personality describes the characteristic behavioral, emotional and cognitive
attributes of an individual.
 Personality is the way of thinking, feeling and behavior that makes a person
different from other people.
 An individual’s personality and tendencies are influenced by experiences
and environment e.g. (surroundings, life situations) and inherited
characteristics.
 A person’s personality typically stays the same over time.
 Biological, genetic, and psychosocial factors during childhood and
adolescence contribute to the development of personality disorders.
 The prevalence of some personality disorders in monozygotic twins is
several times higher than in dizygotic twins.
 It is only when certain personality traits are extreme enough to cause problems
for the person or others that a personality disorder is defined.
 A personality disorder is a way of thinking, feeling and behaving that
deviates from the expectations of the culture, society and causes distress or
problems in functioning and lasts over time.
 Personality disorders are long-term patterns of behavior and inner
experiences that differ significantly from what is expected.
 The pattern of experience and behavior begins by late adolescence or early
adulthood and causes distress or problems in functioning.
 Without treatment, personality disorders can be long-lasting.
 Personality disorders affect and impact at least 2 of these areas (mnemonic=
CAPRI):
 Cognition: Way of thinking about oneself and others
 Affect: Way of responding emotionally
 Personal relations: Way of relating to other people
 Impulse control: Way of controlling one’s behavior
 Although personal distress may occur in some personality disorder, classically
the abnormal personality traits are ‘ego-syntonic’. This is in sharp contrast to
the symptoms in neurotic disorder which are ‘ego dystonic’ and hence cause
significant distress to the patient. So, unlike the patient with neurotic
disorders, several personality disorder patients do not usually seek psychiatric
help unless other psychiatric symptoms co-exist.
 Although personality disorders are usually recognizable by early
adolescences, they are not typically diagnosed before early adult life.
 The symptoms continue unchanged through the adult life and usually become
less obvious in the later years of life (after 40 years of age).

71
 The life-time prevalence of personality disorders in the general population is
about 5-10%.
 Often symptoms of more than one personality disorder are present in one
person. In fact, it is now believed that the occurrence of mixed personality
disorders is commoner than single personality disorders.
 Many patients will meet the criteria for more than one personality disorder,
they should be classified as having all of the disorders for which they qualify.

CLASSIFICATION
 Personality disorder in ICD-10 is divided into 10 types. The validity and
reliability of these categories is limited. Most patients seem to fit several
descriptions or none of them.
 A simpler option, from DSM-5 is to use 3 clusters, which encompass the
individual categories.
 Cluster A (“Eccentric”/ “Weird”): Patients seem eccentric, peculiar or
withdrawn. There is a familial association with psychotic disorders (i.e.
schizophrenia).
o Paranoid- “accusatory”
o Schizoid- “Aloof”
o Schizotypal- “Awkward”
 Cluster B (“dramatic”/ “wild”): Patients seem emotional, dramatic or
inconsistent. There is a familial association with mood disorders.
o Antisocial (dissocial, psychopathic)
o Borderline (emotionally unstable)
o Histrionic
o Narcissistic
 Cluster C (“anxious”/ “worried”): Patients seem anxious or fearful. There
is a familial association with anxiety disorders.
o Avoidant (anxious/ “cowardly”)
o Obsessive compulsive (“compulsive”)
o Dependent (“Clingy”)
CLUSTER A (ECCENTRIC/ WEIRD)
 These are thought to be “odd and eccentric” and on a “schizophrenic-
continuum”
 This cluster includes:
 Paranoid
 Schizoid
 Schizotypal

72
PARANOID PERSONALITY DISORDER (“ACCUSATORY”)
 Patients show a behavior pattern of being suspicious of others and generally
distrustful. They see others as mean or spiteful.
 They often assume that people will harm them or deceive them.
 They tend to blame their own problems on others and seem angry and hostile.
They react severely if they feel they have been lied to or slighted.
 The patients may become involved in litigation on small issues.
 They are often characterized as being pathologically jealous, which leads them
to think that their sexual partners or spouses are cheating on them. They tend
to hold grudges.
 They have superficial relationships and live in isolation as they avoid being
close to people.
 Psychodynamically, the underlying defense mechanism is projection.
 Prevalence: 2-4%. It is more commonly diagnosed in men than in women.
 Treatment:
 Psychotherapy is the treatment of choice (individual psychotherapy and
supportive psychotherapy)
 Patients may also benefit from a short course of antipsychotics for transient
psychosis
 Group psychotherapy should be avoided due to mistrust and
misinterpretation of others’ statements

A 30-year- old man says his wife has been cheating on him because he
does not have a good enough job to provide for her needs. He also
claims that on his previous job, his boss laid him off because he did a
better job than his boss. He has initiated several lawsuits. Refuses
couples’ therapy because he believes the therapist will side with his
wife. Believes neighbors are critical of him.
Diagnosis: Paranoid personality disorder

 Differential diagnosis:
 Delusional (paranoid) disorders
 Paranoid schizophrenia
 Note: unlike patients with schizophrenia, patients with paranoid personality
disorder do not have any fixed delusions and are not frankly psychotic,
although they may have transient psychosis under stressful situations.

73
SCHIZOID PERSONALITY DISORDER (“ALOOF”)
 Patients with schizoid personality disorder have a lifelong pattern of social
withdrawal.
 They are often perceived as eccentric (Strange) and withdrawn.
 They express little emotion or feeling and typically do not seek close
relationships.
 They choose to be alone or enjoy lonely life.
 They seem not to care about praise or criticism.
 They find physical contact less pleasurable e.g. sexual activity or holding
hands.
 They have a flat affect or emotional blunting- they don’t show positive or
negative emotions.
 Prevalence: 3-5%. Diagnosed more often in men than women.
 Treatment:
 Lack insight for individual psychotherapy, and may find group therapy
threatening, may benefit from day programs and drop-in centers.
 Antidepressants if comorbid major depression is diagnosed.

A 45-year- old scientist works in the lab most of the day and has no
friends, according to his coworkers. Has not been able to keep his job
because of failure to collaborate with others. He expresses no desire to
make friends and is content with his single life. He has no evidence of
a thought disorder
Diagnosis: Schizoid personality disorder

 Note: unlike with avoidant personality disorder, patients with schizoid


personality disorder prefer to be alone.
SCHIZOTYPAL PERSONALITY DISORDER (“AWKWARD”)
 Patients have distorted thinking and display eccentric behavior.
 Patients have magical thinking (thinking random events are linked e.g.
because last time I wore a blue shirt it rained, so it will rain today).
 They may have ideas of reference (they think everything directly relates to
their destiny).
 They may have odd, peculiar or strange beliefs.
 They often have peculiar and weird speech.
 They express excessive social anxiety, apprehension or nervousness.
 They are often perceived as strange and odd.

74
 They have few social relations but have strong desires to have them though
they are unable to maintain them.
 Prevalence: 4-5%.
 Treatment:
 Psychotherapy is the treatment of choice to help develop social skills
training
 Short course of low-dose antipsychotics if necessary (for transient
psychosis). Antipsychotics may help decrease social anxiety and suspicion
in interpersonal relationships.

A 35-year- old man dresses in a wizard costume every weekend with


friends as part of a live action role-playing community. He spends a
great deal of time on his computers set up in his basement for video
games and to “detect the presence of extraterrestrial communications in
space.” He has no auditory or visual hallucinations.
Diagnosis: Schizotypal personality disorder

 Schizotypal personality disorder should be differentiated from schizophrenia,


in schizotypal personality disorder the patients are not frankly psychotic
(though they can become transiently so under stress), nor do they have fixed
delusions.
 Note: patients with schizoid personality disorder do not have the same
eccentric behavior seen in patients with schizotypal personality disorder.
CLUSTER B (DRAMATIC/ WILD)
 Cluster B consists of disorders considered “dramatic, emotional and
erratic/impulsive” and a “psychopathic continuum”.
 This cluster of personality disorders has a genetic link with mood disorders
(e.g. depression and bipolar disorder) and substance abuse disorders.
 This cluster includes:
 Antisocial
 Borderline
 Histrionic
 Narcissistic

75
ANTISOCIAL PERSONALITY DISORDER
 These show a pattern of disregarding or violating the rights of others (callous
unconcern).
 They have little empathy, compassion and remorse for their actions.
 They disregard moral values and societal norms.
 They have a low threshold (tolerance) to frustration and a low threshold for
discharge of aggression & irritability.
 They are impulsive, deceitful and often violate the law.
 They may repeatedly lie or deceive others, or may act impulsively i.e. they
can be charming and use this to manipulate others for their personal gain.
 They are frequently skilled at reading social cues and can appear charming
and normal to others who meet them for the first time and do not know
their history
 They are often willing to hurt others if it helps them, this predisposes them to
aggressive and unlawful behavior and at times carrying the label “sociopath”
or “psychopath”.
 This disorder is synonymous with previously used terms such as
psychopathy and sociopathy but does not always mean criminal behavior.
 This type of personality tends to be overrepresented in prison populations and
they show higher rates of substance abuse.
 Individuals must be over 18 years with a history of conduct disorder in order
to meet the diagnosis.
 This disorder is diagnosed more commonly in males.
 The course is usually chronic, however there is some decrease in the
symptoms after the fifth decade of life in some patients.
 Treatment:
 Patients often do not seek psychiatric help and if they do, it is usually under
pressure from the legal authorities.
 The therapeutic alliance is often not sustained.
 Treatment methods:
o Individual psychotherapy
o Psychoanalysis or psychoanalytical psychotherapy
o Group psychotherapy and self-help groups
o Pharmacotherapy: is of little help. Pharmacotherapy may be used to
treat symptoms of anxiety or depression, but use caution due to high
addictive potential of these patients.
 Note: psychotherapy is generally ineffective.

76
A 30-year- old unemployed man has been accused of killing three
senior citizens after robbing them. He is surprisingly charming in the
interview. In his adolescence, he was arrested several times for stealing
cars and assaulting other kids.
Diagnosis: Antisocial personality disorder

BORDERLINE PERSONALITY DISORDER (EMOTIONALLY


UNSTABLE)
 Shows patterns of unstable moods, behaviors and interpersonal relationships.
 They fear abandonment and have poorly formed identity.
 Relationships begin with intense attachments and end with the slightest
conflict.
 Exhibits intense emotional reactions.
 They show an unstable mood (they can quickly go from intense joy to
rage)- this pattern is sometimes called “stable instability” because the only
thing that is consistent is the instability.
 Aggression is common
 Depicts poor self-image and impulsivity.
 They are terrified of abandonment and might do extreme things to avoid
abandonment such as threatening suicide or episodes of self-mutilation.
 Displays inappropriate intense anger and ongoing feelings of emptiness.
 They often use defense mechanism called splitting (important things e.g.
people or a job are seen as either completely good or bad).
 They have higher rates of childhood physical, emotional and sexual abuse than
the general population.
 This is diagnosed 3 times more often in middle aged women than in men.
 There is a high incidence of coexisting major depression and/or substance use
disorders with an increased risk of suicide.
 Treatment:
 Psychoanalysis or psychoanalytical psychotherapy
 Supportive psychotherapy
 Cognitive behavioral therapy (CBT) or dialectical behavior therapy (DBT)
(treatment of choice) approaches or principle have been used with some
success in treatment
 Pharmacotherapy: antidepressants have been used with success in certain
patients with depression. Major depressive episodes, if occurs, necessitates
antidepressant therapy. Occasionally antipsychotics, lithium, valproate or
carbamazepine have been used when aggression or impulsivity are

77
prominent. Pharmacotherapy is not the treatment of first choice in
borderline personality disorder.
A 23-year- old medical student attempted to cut her wrist because
things did not work out with a man she had been dating over the past 3
weeks. She states that guys are jerks and “not worth her time”. She
often feels that she is “alone in this world”.
Diagnosis: Borderline personality disorder

HISTRIONIC PERSONALITY DISORDER


 A pattern of personality with excessive emotion and attention seeking.
 Histrionic personality disorder may quite often be uncomfortable when they
are not the center of attention.
 They may use physical appearance to draw attention to themselves.
 Often they have rapidly shifting or exaggerated emotions. Tantrums or anger
outbursts are common. The actions are not planned for any long-term goals,
instead they seek instant satisfaction and approval.
 They often have superficial relationships: lots of acquaintances and a few deep
relationships because people tend to view them as shallow, flighty and
egocentric.
 This disorder is more common in female gender.
 Hysteria (conversion and dissociation disorder) was previously thought to be
more common in the presence of histrionic personality disorder but recent
studies have failed to prove this relationship.
 Psychodynamically, there are usually intense dependency needs. The defense
mechanism used most often are acting out and dissociation.
 They are often sexually inappropriate and provocative.
 Treatment:
 Psychoanalysis and psychoanalytic psychotherapy are the modes of
treatment which are most successful.
 Pharmacotherapy to treat associated depressive or anxious symptoms as
necessary may be needed.

A 33-year- old scantily clad woman comes to your office complaining


that her fever feels like “she is burning in hell”. She vividly describes
how the fever has affected her work as a teacher.
Diagnosis: Histrionic personality disorder

78
NARCISSISTIC PERSONALITY DISORDER
 They display behavior that attracts need for admiration.
 They lack empathy for others.
 Displays a grandiose sense of self-importance.
 Displays a sense of entitlement
 Takes advantage of others.
 Despite these features they present with, they have a fragile self-esteem
(vulnerable to criticism and lash out when feeling slighted).
 They come across as pretentious, self-centered and entitled often exploiting
others as they only get involved in situations that advance personal agenda.
 There is a higher incidence of depression and midlife crises since these
patients put such a high value on youth and power.
 Treatment:
 Psychotherapy is the treatment of choice.
 Antidepressants may be used if a comorbid mood disorder is diagnosed.

A 33-year- old company CEO is rushed to the ED after an automobile


accident. He does not let the residents operate on him and requests the
chief of trauma surgery because he is “vital to the company”. He makes
several business phone calls in the ED to stay on stay on “top of his
game”.
Diagnosis: Narcissistic personality disorder

CLUSTER C (ANXIOUS/WORRIED)
 Cluster C has disorders considered “anxious and fearful” and characterized by
“introversion”.
 They include
 Avoidant
 Obsessive compulsive
 Dependent
AVOIDANT PERSONALITY DISORDER
 Disposition of a pattern of extreme shyness. They are shy, timid and socially
inhibited with low self-esteem.
 They view themselves as incapable, inadequate and undesirable.
 They often want close relationships but rarely take social risks and avoid
social situations making it hard for them to meet people.
 They are hypersensitive to rejection and criticism becoming even more
withdrawn when they happen.

79
 They only accept involvement if they are certain of being liked.
 They are often preoccupied with being criticized or rejected.
 May view themselves as not being good enough or socially incompetent.
 There is a considerable overlap between avoidant personality and social
phobias. One key difference is social phobias focus on anxiety of specific
situations e.g. public speaking while avoidant personality is associated with
anxiety of general situations.
 Avoidant personality disorder is equally frequent in males and females.
 There is an increased incidence of associated anxiety and depressive disorders.
 Treatment:
 Individual psychotherapy
 Group therapy
 Behavior therapy: in particular, social skills training and assertiveness
training are useful.
 CBT: the focus is on negative thoughts and negative self-appraisal.
 Selective serotonin reuptake inhibitors (SSRIs) may be prescribed for
comorbid social anxiety disorder or major depression.

A 30-year- old postal worker rarely goes out with her coworkers and
often makes excuses when they ask her to join them because she is
afraid they will not like her. She wishes to go out and meet new people
but, according to her, she is too “shy”.
Diagnosis: Avoidant personality disorder

OBSESSIVE COMPULSIVE PERSONALITY DISORDER


 Individuals are obsessed with orderliness, perfection and control.
 They are overly focused on details or schedules.
 May work excessively not allowing time for leisure or friends.
 May be inflexible in their morality and values. They are rigid with beliefs and
moral issues and are often perceived as stubborn.
 They are easily stressed and surprisingly inefficient because they spend so
much time planning and worrying instead of actually doing the task.
 This is not the same as obsessive compulsive disorder. There is however some
overlap between obsessive compulsive disorder (OCD) and obsessive
compulsive personality disorder (OCPD). OCD is an anxiety disorder
associated with repetition of ritualistic actions e.g. repeated checking if the
door is locked.

80
 OCD is ego-dystonic because the person wishes they could stop the behavior
however OCPD is ego-syntonic because the person is generally happy with
how they are and don’t want to change anything about them.
 Men are two times more likely to have OCPD than women.
 Treatment:
 Psychotherapy is the treatment of choice.
o Cognitive-behavior therapy may be particularly useful.
o Group psychotherapy.
 Pharmacotherapy may be used to treat associated symptoms as necessary.

A 40-year- old secretary has been recently fired because of her inability
to prepare some work projects in time. According to her, they were not
in the right format and she had to revise them six times, which led to
the delay. This has happened before but she feels that she is not given
enough time.
Diagnosis: Obsessive-compulsive personality disorder

DEPENDENT PERSONALITY DISODER


 A pattern of needing to be taken care of and submissive, clingy behavior.
 Individuals have intense fear of separation and rejection and tend to overly
depend on or cling to the relationships they have.
 They lack self-confidence and can’t care for themselves.
 People with dependent personality disorder may have difficult making daily
decisions without reassurance from others or may feel uncomfortable or
helpless when alone because of the fear of inability to take care of themselves.
 They are often trapped in abusive relationships.
 Women are more likely to be diagnosed than men.
 Treatment:
 Individual psychotherapy
 Group psychotherapy
 Behavior therapy (such as assertiveness training and social skills training)
is often useful.
 CBT: the focus is on negative thoughts and negative self-appraisal.
 Pharmacotherapy may be used to treat associated symptoms of anxiety or
depression.

81
A 40-year- old man who lives with his parents has trouble deciding
how to get his car fixed. He calls his father at work several times to ask
very trivial things. He has been unemployed over the past 3 year.
Diagnosis: Dependent personality disorder

DIAGNOSIS
 According to ICD-10, the diagnostic guidelines for specific personality
disorder include conditions not directly attributable to gross brain damage or
disease, or to another psychiatric disorder, meeting the following criteria:
1. Markedly disharmonious attitudes and behavior involving usually several
areas of function e.g. Cognition, Affect, Personal relations, and Impulse
control.
2. The abnormal behavior pattern is enduring of longstanding and not limited
to episodes of mental illness.
3. The abnormal behavior pattern is pervasive and clearly maladaptive to a
broad range of personal and social situations.
4. The manifestations always appear during childhood or adolescence and
continue into adulthood. People under 18 are typically not diagnosed with
personality disorder because their personalities are still developing.
5. The disorder leads to considerable personal distress but this may only
become apparent late in its course.
6. The disorder is usually but not invariable, associated with significant
problems in occupational and social performance.

TREATMENT
 Personality disorders are generally very difficult to treat, especially since few
patients are aware that they need help. The disorders tend to be chronic and
lifelong.
 In general, pharmacologic treatment has limited usefulness except in treating
comorbid mental conditions e.g. major depressive disorder
 There are no medications specifically to treat personality disorders.
However, medication, such as antidepressants, antianxiety medication or
mood stabilizing medication, may be helpful in treating some symptoms.
 More severe or long lasting symptoms may require a team approach
involving a primary care provider or doctor, a psychiatrist, a psychologist,
social worker and family members.

82
 Psychotherapy is usually the most helpful.
 During psychotherapy, an individual can gain insight and knowledge about
the disorder and what is contributing to symptoms and can talk about
thoughts, feelings and behaviors.
 Psychotherapy can help a person understand the effects of their behavior
on others and learn to manage or cope with symptoms.
 Psychotherapy can help to reduce behavior causing problems with function
and relationships.
 Commonly used types of psychotherapy:
o Psychoanalytic/Psychodynamic therapy
o Dialectical behavior therapy
o Cognitive behavioral therapy
o Group therapy
o Psychoeducation (teaching the individual and family members about
the illness, treatment and ways of coping)
 Psychodynamic approach:
 Some examples of assumptions that drive the psychodynamic approach
are:
o The unconscious is one of the most powerful effects on behavior and
emotion.
o No behavior is without cause and is therefore determined.
o Childhood experience greatly affect emotions and behavior as adults.
o The psychodynamic approach includes all the theories in psychology
that see human functioning based upon the interaction of drives and
forces within the person.
o They focus particularly on the unconscious and between the different
structures of the personality.
o Sigmund Freud’s psychoanalysis is both a theory and therapy.
 Psychoanalysis is also known as “talk therapy”.
 Psychoanalysis and psychodynamic therapies.
 This approach focuses on changing problematic behaviors, feelings, and
thoughts by discovering their unconscious meanings and motivations e.g.
in Behavior therapy, Cognitive therapy, Humanistic therapy and
Integrative or holistic therapy
 Dialectical behavior therapy: Dialectical behavior therapy is an evidence-
based psychotherapy that began with efforts to treat Borderline Personality
disorders. DBT has been proven useful in treating mood disorders, suicidal
ideation and for change in behavioral patterns such as self-harm and substance
abuse.

83
FACTITIOUS DISORDER (MUNCHAUSEN
SYNDROME)
 Munchausen syndrome (also known variously as hospital addiction, hospital
hobo syndrome, peregrinating (wandering) problem patients, polysurgical
addiction, hospital vagrant or professional patients) is used for those patients
who repeatedly simulate or fake disease for the sole purpose of obtaining
medical attention.
 There is no other recognizable motive (hence, it is different from malingering).
 Patients with factitious disorder intentionally fake medical or psychological
signs and symptoms in order to assume the role of a sick patient.
 Factitious disorder is more common in women. It accounts for at least 1% of
hospitalized patients with a higher incidence in hospital and health care
workers (who have learned how to feign symptoms).
 Factitious disorder is associated with personality disorders and many patients
have a history of illness and hospitalization, as well as childhood physical or
sexual abuse i.e. deprivation and neglect.
 The cause is not clear. Probably these patients are masochistic, seek
dependency from a father-figure (e.g. the physician), attempt to manoeuver
control over the father-figure and see the surgical procedure as partial suicide.

CLINICAL FEATURES
 Factitious disorders can present with predominantly physical signs and
symptoms, or psychological signs and symptoms or combined signs and
symptoms.
 The patients distort their clinical histories, laboratory test reports and even
facts about other aspects of their lives (pseudologia fantastica).
 Sometimes they distort physical signs by self-inflicted injuries and secondary
infections. Drug abuse especially abuse of prescription drugs is common.
 Evidence of earlier treatment, usually surgical procedures is often available in
the form of multiple scars (e.g. grid-iron abdomen).
 These patients are often manipulative and convincingly tell lies, create
problems in the inpatient setting and often leave against medical advice
usually after the surgical procedure has been performed.
 Psychological signs and symptoms:
 Patients with factitious disorders may simulate psychological conditions
and psychiatric disorder.

84
 A patient may feign/simulate/pretend bereavement by reporting that
someone to whom he or she was close has died or been killed in an
accident.
 A patient may simulate symptoms of posttraumatic stress disorder.
 Provide false reports of previous trauma (e.g. a civilian accident or combat
experience)
 Closely related to factitious posttraumatic stress disorder is the false
victimization syndrome, in which the patient falsely claims some type of
abuse. A woman may falsely report that she had been raped.
 Other simulated psychological disorder includes various forms of
dementia, amnesia, or fugue, multiple personality disorder and more rarely
schizophrenia.
 Physical signs and symptoms:
 The production of physical symptoms or disease is probably the most
common form of factitious disorder.
 Essentially medical diseases and symptoms have been either simulated or
artificially produced at one time or another.
 Among the most common of these disorders are:
o Factitious hypoglycemia
o Factitious anemia
o Factitious gastrointestinal bleeding
o Pseudoseizures
o Simulation of brain tumors
o Simulation of renal colic
o More recently simulation of AIDS
 Combined psychological and physical signs and symptoms:
 A patient may be admitted to the hospital with factitious physical
symptoms. In the course of hospitalization, perhaps in an attempt to obtain
more sympathy or interest, may report or simulate a variety of
psychological symptoms.
 Having experienced the recent loss of close relative or friend or having
been raped in the past.
 Factitious disorder not otherwise specified:
 This category is reserved for forms of factitious disorder that do not fit one
of the other categories.
 It includes the Munchausen syndrome by proxy, in which one person
secretly induces disease or reports disease in another person.
 Most commonly, this is the behavior of a mother in reference to the child
or baby

85
DIAGNOSIS
 Falsification of physical or psychological signs or symptoms or induction of
injury or disease, associated with identified deception.
 The deceptive behavior is evident even in the absence of obvious external
rewards (such as in malingering).
 Behavior is not better explained by another mental disorder such as delusional
disorder or another psychotic disorder.
 Individual can present him/herself or another individual (as in factitious
disorder imposed on another/ Munchausen syndrome by proxy)
 Munchausen syndrome by proxy is intentionally producing symptoms in
someone else who is under one’s care (usually one’s children)
 Commonly feigned symptoms:
 Psychiatric- hallucinations, depression
 Medical- fever (by heating the thermometer), infection, hypoglycemia,
abdominal pain, seizures and hematuria

TREATMENT AND PROGNOSIS


 Collect collateral information from medical treaters and family.
 Collaborate with primary care physician and treatment team to avoid
unnecessary procedures.
 Patients may require confrontation in a nonthreatening manner, however
patients who are confronted may leave against medical advice and seek
hospitalization elsewhere.
 Repeated and long-term hospitalizations are common.
 The prognosis is usually poor and treatment often unsuccessful.
 Certain points must be kept in mind by the physician (or surgeon) after the
diagnosis has been made:
 Avoid feelings of anger, hostility and ridicule which are aroused by the
discovery of factitious illness.
 Patients should not be confronted or labelled as liars. Instead, a psychiatric
or psychological consultation should be sought as these patients may need
help.
 Of course, the unnecessary surgical procedure(s) should not be carried out.
 One approach to handling a patient with a factitious disorder is therapeutic
double bind or contingency management.
 It involves informing the patient that a factitious disorder may exist.
 The patient is further told that failure to respond fully to medical care
would constitute conclusive evidence that the patient’s problem is not
organic but rather psychiatric.

86
 The problem is therefore reframed or redefined in such a way that,
symptoms and their resolution are both legitimized.
 The patient has little choice but to accept and respond to a proposed course
of action or seek care elsewhere.

IMPLICATIONS
 These patients are willing to undergo incredible hardship:
 Limb amputation
 Organ loss
 Even death to perpetuate the masquerade
 Although multiple hospitalizations often lead to:
 Iatrogenic physical conditions
 Postoperative pain syndromes
 Drug addictions
 Patients continue to crave hospitalization for its own sake
 The patients typically have a fragile and fragmented self-image and are
susceptible to psychotic and even suicidal episodes.
 Identification of a factitious disorder is usually made in 1 of 4 ways:
 The patient is accidentally discovered in the act
 Incriminating items are found
 Laboratory values suggest non-organic etiology
 The diagnosis is made by exclusion

MALINGERING
 Malingering involves the intentional reporting of physical or psychological
symptoms in order to achieve personal gain.
 Common external motivations include the avoidance of military conscription
or duty, avoidance of work, police, receiving room and board, obtaining
narcotics, and receiving monetary compensation.
 Not that malingering is not considered to be a mental illness.
 It is not uncommon in hospitalized patients.
 Significantly it is commoner in men than women.

CLINICAL FEATURES
 Patients usually present with multiple vague complaints that do not conform
to a known medical condition.
 They often have a long medical history with many hospital stays.
 They are generally uncooperative and refuse to accept a good prognosis even
after extensive medical evaluation.

87
 Their symptoms improve once their desired objective is obtained.
 Amnesia is the most common psychological presentation. Followed by
paranoia, morbid depression, suicidal ideation and psychosis.
 The person exhibits or has a history of antisocial behavior.
 Treatment is not required as this is not a psychiatric condition.

A 37-year-old patient claims that he has frequent episodes of “seizures”,


starts on medication and joins an epilepsy support group. It becomes
known that he is doing this in order to collect social security disability
money.
Diagnosis= MALINGERING

 Somatic symptoms disorders: patients believe they are ill and do not
intentionally produce or feign symptoms.
 Factitious disorder: patients intentionally produce symptoms of a
psychological or physical illness because of a desire to assume the sick
role, not for external rewards.
 Malingering: patients intentionally produce or feign symptoms for
external rewards.

88
SEXUAL PERVERSIONS AND PARAPHILIAS
 Sexual perversion or paraphilia is a condition in which a person’s sexual
arousal and gratification depend on fantasizing about and engaging in sexual
behavior that is atypical and extreme.
 Paraphilias (Sexual deviations, perversions) are disorders of sexual preference
in which sexual arousal occurs persistently (at least 6 months) and
significantly in response to objects which are not a part of normal sexual
arousal
 A paraphilia can revolve around a particular object e.g. nonhuman objects,
suffering or humiliation of self and/or sexual partner, children or
nonconsenting person.
 It can be a particular behavior such as inflicting pain, exposing oneself.
 A paraphilia is considered a disorder when it causes distress or threatens to
harm someone else.
 Paraphilic fantasies alone are not considered disorders unless they are intense,
recurrent and interfere with daily life, occasional fantasies are considered
normal components of sexuality (even if unusual).
 Most paraphilic disorders occur almost exclusively in men, but sadism,
masochism and pedophilia may also occur in women.
 Voyeurism, pedophilia and exhibitionism are the most common paraphilic
disorders.
 Patients often have more than one paraphilia.

ETIOLOGY
 It is unclear what causes paraphilic disorders to develop.
 Psychoanalysts theorize that an individual with a paraphilia is repeating or
reverting to a sexual habit that arose early in life.
 Behaviorists suggest that paraphilias begin through a process of conditioning
 Nonsexual objects can become sexually arousing if they are repeatedly
associated with pleasurable sexual activity.
 Or particular sexual acts (such as peeping, exhibiting, bestiality) that
provide especially intense erotic pleasure can lead the person to prefer that
behavior
 In some cases, there seems to be a predisposing factor such as difficulty
forming person-to-person relationships.
 Behavioral learning models suggest that a child who is the victim or
observer of inappropriate sexual behaviors may learn to imitate that
behavior and is later reinforced for it.

89
 Compensation models suggest that these individuals are deprived of
normal social sexual contacts and thus seek gratification through less
socially acceptable means.

COMMON PARAPHILIAS
 Common paraphilias include:
 Pedophilia
 Exhibitionism
 Voyeurism/scoptophilia
 Frotteurism
 Fetishism
 Sexual sadism
 Sexual masochism
 Fetishistic transvestism (Transvestic disorder)
 Zoophilia (bestiality)
 Other paraphilas include sexual arousal with urine (urophilia), feces
(coprophilia), enemas (klismaphilia), corpses (necrophilia), among many
others.
 Some of the behaviors associated with paraphilia are illegal. Individuals under
treatment for paraphilic disorder often encounter legal complications
surrounding their behaviors.
PEDOPHILIA
 Pedophilia is a persistent or recurrent involvement of an adult (age> 16 years)
and at least 5 years older than the child in sexual activity with prepubertal
children (age 13 years or younger) either heterosexual or homosexual.
 This may be associated with sexual sadism.
 The pedophilic behavior may be either limited to incest or may spread to
children outside the family.
 In most civilized societies, pedophilia is a serious offense and the convicted
pedophiles’ name remains on a sex offenders’ register in order to protect the
society.
PEDERASTY
 Sexual activity involving a young male.

90
EXHIBITIONISM
 This is a persistent (or recurrent) and significant method of sexual arousal by
the exposure of one’s genitalia to an unsuspecting stranger.
 This is often followed by masturbation to achieve orgasm.
 The disorder is almost exclusively seen in males, and the ‘unsuspecting
stranger’ is usually a female (child or adult).
 Two groups are recognized:
 Type 1: Inhibited men who struggle against the urge, feel guilty and expose
a flaccid penis.
 Type 2: Aggressive men who expose an erect penis and masturbate then or
later
 Type 2 show an association with actual sexual assaults. Behavioral treatments
may be beneficial. SSRIs have a limited evidence base.
VOYEURISM/SCOPTOPHILIA
 This is a persistent or recurrent tendency to observe unsuspecting persons
(usually of the other sex) naked, disrobing or engaged in sexual activity.
 This is often followed by masturbation to achieve orgasm without the
observed person(s) being aware.
 This is almost always seen in males.
 Watching pornography is not included here.
FROTTEURISM
 This is a persistent or recurrent involvement in the act of touching and rubbing
against an unsuspecting, nonconsenting person (usually of the other sex).
 Frottage is often employed in crowded places e.g. buses, where the victim
does not protest because she cannot suspect that frottage can be committed
there.
 This is often seen in adolescent males.
FETISHISM
 In fetishism, the sexual arousal occurs either solely or predominantly with a
nonliving object which is usually intimately associated with the human body.
 The word fetish means magical i.e. the non-living object ‘magically’ become
phallic for that person.
 Fetishism is not diagnosed if the sexual object is the wearing of clothes of
opposite sex (Fetishistic transvestism), the use of a human body part
(masturbation), or the use of a genital-stimulating object (e.g. vibrator).
 Fetishism is very often associated with masturbation.

91
 An example of fetishistic disorder is a man being primarily sexually aroused
by woman’s shoes causing significant distress and marital problems.
SEXUAL SADISM
 In this disorder, the person (the ‘sadist’) is sexually aroused by physical and/or
psychological humiliation, suffering or injury of the sexual partner (the
‘victim’).
 Most often the person inflicting the suffering is male, although this is not
essential.
 The methods used range from restraining by tying, beating, burning, cutting,
stabbing, to rape and even killing.
SEXUAL MASOCHISM
 This is just the reverse of sexual sadism.
 Here the person (the ‘masochist’) is sexually aroused by physical and/or
psychological humiliation, suffering or injury inflicted on self by other
(usually ‘sadists’).
 Most often the masochist is a female though any pattern is possible.
 The methods used are the same as the ones used in sexual sadism. Only there
is a role reversal.
 To be called a disorder, this should be persistent and significant mode of
sexual arousal in the person.
 Sexual sadism and masochism are often seen in the same individual and are
on a continuum, therefore they are classified together as sadomasochism in
ICD-10.
FETISHISTIC TRANSVESTISM (TRANSVESTIC DISORDER)
 This disorder occurs exclusively in heterosexual males.
 The person actually or in fantasy wears clothes of the opposite sex (cross-
dressing) for sexual arousal.
 An example of transvestic disorder is a person being sexually aroused by
dressing up as a member of the opposite gender. This does not mean they are
homosexual.
 This disorder should be differentiated from dual-role transvestism and
transsexualism.
 This disorder may be associated with fantasies of other males approaching the
person who is in a female dress.
 Masturbation or rarely coitus is associated with cross-dressing to achieve
orgasm.

92
 To be called a disorder, this should be a persistent and significant mode of
sexual arousal in the person.
ZOOPHILIA (BESTIALITY)
 Zoophilia as a persistent and significant involvement in sexual activity with
animals is rare.
 Occasional or situational zoophilia is much more common.

TREATMENT
 Paraphilias are difficult to treat.
 Treatment approaches include: traditional psychoanalysis, hypnosis, behavior
therapy techniques and drugs.
 Psychoanalysis and psychoanalytic psychotherapy: This is of particular help
if the patient is psychologically minded and has good ego strength for therapy.
 Behavior therapy: aversion therapy is the treatment choice in severe,
distressing paraphilia with the patient’s consent.
o Aversion conditioning: for example, involves using negative stimuli to
reduce or eliminate a behavior.
 Assisted aversive conditioning is similar to covert sensitization
except the negative event is made real, most likely in the form of
a foul odor pumped in the air by the therapist.
 The goal is for the patient to associate the deviant behavior with
the foul odor and take measures to avoid the odor by avoiding the
said behavior.
o Covert sensitization: entails the patient relaxing, visualizing scenes of
deviant behavior followed by a negative event, such as getting his penis
struck in the zipper of his pants.
o Vicarious sensitization: entails showing videotapes of deviant behavior
and their consequences, such as victims describing desired revenge or
perhaps even watching surgical castrations.
o There are also positive conditioning approaches that might center on
social skills training and alternate behaviors the patient might take that
are more appropriate.
o Reconditioning techniques center on providing immediate feedback to
the patient so that the behavior will be changed right away.
 For example, a person might be connected to a plethysmographic
biofeedback machine that is hooked up to a light. The person is
taught to keep the light within a specific range of color while the
person is exposed to sexually stimulating material.

93
o Masturbation training might focus on separating the pleasure inherent
in masturbation and climax from the deviant behavior.
 Drug therapy:
 Antipsychotics have sometimes been used for severe or dangerous
aggression associated with paraphilias. Benperidol was earlier believed to
be particularly useful but the claim has not been substantiated, and the drug
is not available in the market.
 Antiandrogens (cyproterone acetate or medroxy-progesterone acetate-
Depo-provera) can be used in paraphilias with excessive sexual activity.
o This class of drugs drastically lower testosterone levels temporarily.
o This results in reduction of frequency of erections, sexual fantasies and
initiation of sexual behaviors including masturbation and intercourse.
o The drug lowers sex drive in males and can reduce the frequency of
mental imagery of sexually arousing scenes. This allows for
concentration on counseling without a strong distraction from the
paraphilic urges.
o Note: the level of an individual’s sex drive is not consistently related to
paraphiliac behavior. Additionally, high levels of circulating
testosterone do not predispose a male to paraphilias.
o Increasingly the evidence suggests that combing drug therapy with
cognitive behavior therapy can be effective.
 Antidepressant such as fluoxetine (Prozac) have also successfully
decreased sex drive but have not effectively targeted sexual fantasies.
 Cognitive therapy: includes restructuring cognitive distortions and empathy
training.
 Restructuring cognitive distortions:
o Involves correcting erroneous beliefs by the patient, which may lead to
errors in behavior such as seeing victim and constructing erroneous
logic that the victim deserves to be part of the deviant act.
 Empathy training:
o Involves helping the offender take on the perspective of the victim and
better identify with them, in order to understand the harm that has been
done.
 Other treatment: castration and psychosurgery are extremely rare choices
these days.

94
SUBSTANCE MISUSE
 A substance is regarded as being misused (or abused) if it produces physical,
psychological or social harm.
 Substance use disorder is characterized by problematic pattern of use
(impairment or distress manifested by at least 2 of the following within a year,
regardless of the substance):
 Using substance more than originally intended
 Persistent desire or unsuccessful efforts to cut down on use
 Significant time spent in obtaining, using or recovering from substance
 Craving to use substance
 Failure to fulfil obligations at work, school or home
 Continued use despite social or interpersonal problems due to the substance
use
 Decreased social, occupational or recreational activities because of
substance use
 Use in dangerous situations (e.g. driving a car)
 Continued use despite subsequent physical or psychological problem (e.g.
drinking alcohol despite worsening liver problems)
 Tolerance
 Withdrawal
 Substance abuse is common in men than women.
 Commonly misused substances include:
 Alcohol
 Prescribed drugs and legal drugs: Benzodiazepines, nicotine, caffeine and
cannabis
 Opioids: heroin, morphine, pethidine, methadone
 Stimulants: amphetamines, cocaine, ecstasy
 Hallucinogens: LSD, phencyclidine, solvents
 Substance misuse presents in diverse ways for example as depression or
morbid jealousy (both associated with alcohol), acute psychosis
(amphetamines) or as hematemesis (alcoholic cirrhosis).
 There are several categories of substance misuse:
 At risk consumption (alcohol) intake at a level associated with increased
risk of harm.
 Harmful use- misuse associated with health and social consequences but
without dependence.
 Dependence- prolonged, regular use of some substances (especially
alcohol, opioids, amphetamines) can lead to dependence (addiction) and
withdrawal syndromes.

95
 Intoxication-is the acute effect of the substance- being drunk (alcohol),
tripping (LSD) or stoned (cannabis). Intoxication with illicit drugs may
lead to acute psychiatric presentations.
 Etiological factors associated with substance misuse are multifactorial and
include genetic, neurobiological, psychological, socioeconomic and legal
factors.
 Mood symptoms are common among persons with substance use disorders.
 Note: Substance-induced mood symptoms improve during abstinence,
whereas primary mood symptoms persist.
 Psychotic symptoms may occur with some substances.
 Personality disorders and psychiatric comorbidities (e.g. major depression,
anxiety disorders) are common among persons with substance use disorders.
 It is often challenging to decide whether psychiatric symptoms are primary or
substance induced.
 Withdrawal: the development of a substance-specific syndrome due to the
cessation (or reduction) of substance use that has been heavy and prolonged.
 Tolerance: the need for increased amounts of the substance to achieve the
desired effects or diminished effect if using the same amount of the substance.
 Withdrawal symptoms of a drug are usually opposite of its intoxication
effects. For example, alcohol is sedating, but alcohol withdrawal can lead
brain excitation and seizures.
 Both the intoxicated and withdrawing patient can present difficulties in
diagnosis and treatment. Since it is common for persons to abuse several
substances at once, the clinical presentation is often confusing and
signs/symptoms may be atypical.
 Treatment of substance use disorders:
 Behavioral counseling should be part of every substance use disorder
treatment.
 Psychosocial treatments are effective and include:
o Motivational intervention (MI)
o Cognitive behavioral therapy (CBT)
o Contingency management
o Individual therapy
o Group therapy
 Twelve-step groups such as Alcoholics anonymous (AA) and Narcotics
Anonymous (NA) should also be encouraged as part of the treatment.
 Pharmacotherapy is specific for some drugs of abuse.

96
ALCOHOL
 Ethyl alcohol is a natural product of breakdown of carbohydrates in plants.
 Its euphoriant and intoxicating properties have been known from prehistoric
times.
 There has been an increase in drinking ever since and control measures put in
place but alcohol is still being consumed.
 In the past, primary prevention of alcohol and drug use was:
 Controlling availability
 Ensuring resistance to individuals by way of education
 Persuasion
 Alternative provision- provision of bars/number of hours etc.
 Laws- 18 years and above/21 years etc.
 Secondary prevention: these are already drinking and the aim is to prevent
further damage such as disease or social function loss.
 The harm depends on: age, gender, setting, culture, genetic make-up, pattern
of consumption and amounts in units.
EPIDEMIOLOGY
 1 unit= 8g of ethanol= half a pint of 3.5% alcohol volume bear or a glass of
125mls, Tot of spirits
 In Spain/Australia 1 drink contains 10g ethanol, in the USA 1 drink contains
13g ethanol.
% 𝑎𝑙𝑐𝑜ℎ𝑜𝑙 ×𝑣𝑜𝑙𝑢𝑚𝑒
 Units of alcohol=
1000
 Mosi lager units of alcohol= (4% x 375ml)/1000= 1.5 units
 Prevalence of alcohol related problems equals alcohol consumption per
person.
 High consumption of alcohol leads to high morbidity (cirrhosis). Less
sporadic drinking is less harmful than bout/binge drinking patterns.
 Drinking by women has gone up from the 20th century.
 Drinking is high in young age groups.
 Ethnicity and religious minorities have different drinking patterns; Hindus/
Sikhish/ SDA/ Baptists oppose/prohibit drinking.
 In the US/UK heavy drinkers are among the Indians, Pakistanis.
 Occupations lead to heavy drinking.
 Availability of alcohol at the work place.
 Social expectations (business meetings)
 Separation from normal, social and sexual relationships

97
 High income: doctors, lawyers, senior executives are all among heavy
consumers.
DEPENDENCY/ADDICTION
 A cluster of physiological, behavioral and cognitive phenomena in which the
use of a substance or class of substances takes on a higher priority for a given
individual than other behavior that once had great value.
 According to the ICD for a dependency syndrome to be diagnosed 3 or more
of the following should be present:
 Strong sense of compulsion to take substance
 Difficulty in controlling substance taking behavior in terms of onset,
termination or level of use
 A physiological withdrawal state when substance has been ceased or
reduced
 Evidence of tolerance
 Progressive neglect of alternative pleasure or interests
 Persistent substance use despite clear overtly harmful consequences
EFFECTS OF ALCOHOL
 Alcohol (ethanol) activates gamma-aminobutyric acid (GABA), dopamine
and serotonin receptors in the CNS and inhibits glutamate receptor activity
and voltage-gated calcium channels.
 GABA receptors are inhibitory and glutamate receptors are excitatory.
Thus alcohol is a potent CNS depressant.
 Alcohol is metabolized in the following manner:
 Alcohol acetaldehyde (enzyme: alcohol dehydrogenase)
 Acetaldehyde  acetic acid (enzyme: aldehyde dehydrogenase)
 There is an upregulation of these enzymes in heavy drinkers.
 Secondary to a gene variant, Asians often have less aldehyde dehydrogenase,
resulting in flushing and nausea, and likely reducing their risk of alcohol use
disorder.
 Alcohol related social harm involves family relationships, economic factors,
employment problems, crime, and drunkesnness offences.

98
MANAGEMENT OF ALCOHOL RELATED PROBLEMS
 History: always take a good history inclusive of quantities, problems
associated, dependence, withdrawal, harm, social history, personal history and
other drug use.
 Examination- cutaneous and superficial signs
 Spider naevi, talengiectasia, facial mooning, parotid enlargement, palmer
erytherma, dupytrens contracture, gynecomastia etc.
 Labs- blood tests (MCV increases), U&Es, albumin, LFTs, triglycerides,
clotting time
 Pharmacotherapy
 This is medication to minimize withdrawal symptoms:
o Admit after assessment
o Detoxification
o Benzodiazepines- Chlordiazepoxide 80-100mg/ day divided doses,
Diazepam can also be used
o Chlormethiazole effectively controls withdraw symptoms, however not
safe for outpatient. Alcohol and Chlormethiazole interaction causes
death- respiratory depression.
o Patients are discharged when fits have gone, feed well and confusion is
gone.
o Vitamin supplementation can go on for several months.
 Disulfiram: inhibits aldehyde dehydrogenase and leads to accumulation of
alcohol metabolites i.e. acetaldehyde.
o Dosage- 200mg-800 mg per day
o Acetaldehyde accumulates in blood and causes discomfort: flushing,
headache, nausea, hypotension, labored breath.

99
o It should be given when alcohol breath test is zero and cannot be given
to persons with: heart disease, suicidal tendencies, hypotension
treatment and active liver disease.
 Acamprosate: this is a GABA agonist and Glutamate antagonist
o More tolerable
o Dose: 666mg TDS 6/12 or 2g divided doses
o It causes diarrhea, irregular heartbeat, head aches
o It is not indicated in patient above 65 years.
 Naltrexone: this is a GABA agonist and glutamate antagonist as well as an
opiate antagonist
o It blocks euphoric effects
o It is not dose dependent
o This drug limits the amounts one can drink then it gives nausea,
dizziness, headache, insomnia
 Nelmefene/nelmetrene/silincro: similar effects to naltrexone.
 Antidepressants: fluoxetine, fluvoxamine.
 Note: after detox insomnia is experienced and patients may relapse as such
give trazedone a GABA agonist and muscle relaxant.
 Psychological management:
 Social skills training
 CBT
 Group therapy
 Co joint and family therapy
 Community and reinforcement
 Approach social network behavioral therapy
 Alcohol advice center
 Employment policies
 Alcoholics anonymous
 Specialized centers.
INTOXICATION
CLINICAL PRESENTATION
 Absorption and elimination rates of alcohol are variable and dependent on
many factors including age, sex, body weight, chronic nature of use, duration
of consumption, food in the stomach and the state of nutrition and liver health.
 The effects of ethanol also depend on blood alcohol level (BAL).
 Serum alcohol level or an expired air breathalyzer can determine the extent
of intoxication.

100
 Effects of alcohol:
 Decreased fine motor control
 Impaired judgement and coordination
 Ataxic gait and poor balance
 Lethargy, difficulty sitting upright, difficulty with memory, nausea/
vomiting
 Rigidity of thought
 Coma in the novice drinker
 Respiratory depression and death

Figure 4: Clinical presentation of Alcohol intoxication

 Factors causing the effects of alcohol include:


 Thiamine deficiency (Wernicke’s encephalopathy)
 Head injury
 Alcohol withdrawal fits
 Dementia
 Cerebellar degeneration, cerebellar pontine degeneration
 Marchiafava Bignami syndrome- progressive neurological disease of
alcoholism- corpus callosum demyelination and necrosis and subsequent
atrophy

101
TREATMENT OF INTOXICATION
 History and examination
 Laboratory investigations-
 Serum: blood glucose, urea and electrolytes, LFTs
 Imaging: MRI/CT scan, note any white matter cortical atrophy or
ventricular enlargement
 Monitor: airway, breathing, circulation.
 Severely intoxicated patient may require mechanical ventilation with
attention to acid-base balance, temperature and electrolytes while he or she
is recovering.
 Give thiamine (to prevent Wernicke’s encephalopathy) and folate.
 Naloxone may be necessary to reverse effects of co-ingested opioids.
 Computed tomographic (CT) scan of the head may be necessary to rule out
subdural hematoma or other brain injury.
 The liver will eventually metabolize alcohol without any other interventions.
 Gastrointestinal evacuation (e.g. gastric lavage, induction of emesis and
charcoal) is not indicated in the treatment of alcohol (Ethanol) overdose unless
a significant amount of alcohol was ingested within the preceding 30-60
minutes.
 Alcohol detoxification- disulfiram (when alcohol breath is zero), acamprosate,
naltrexone

WITHDRAWAL AND DELIRIUM TREMENS


 Chronic alcohol use has a depressant effect on the CNS and cessation of use
causes a compensatory hyperactivity.
 Alcohol withdrawal is potentially lethal!
 The main features of alcohol withdrawal are:
 Tremulousness (‘The shakes’)
 Agitation
 Nausea and retching
 Sweating
 Overwhelming desire to drink (craving).
 Withdrawal symptoms are relieved by alcohol. If untreated the symptoms may
last for several days. Transient misperception and hallucinations may occur.
 Withdrawal symptoms often occur on waking as the blood alcohol
concentration falls during sleep
 The most severe forms of withdrawal (5% of cases) is delirium tremens (the
‘DTs’) a potentially fatal conditions

102
 Delirium tremens is present in 5% of cases
 Onset 24-48 hours after stopping heavy prolonged drinking
 It is characterized by
o Coarse tremors in the limbs
o Restlessness
o Loss of contact with reality: visual hallucination, persecutory and
paranoid delusions with persecutory
o Fear and agitation sometimes aggression
o Delirium and clouding of consciousness with disorientation
o Seizures
o Autonomic disturbance (sweating, fever, tachycardia, hypertension)
o Insomnia
o Dehydration and electrolyte disturbance
o Shock
o Wernicke’s encephalopathy
o Hypoglycemia
 These symptoms lasts 3-4 days, followed by exhaustion and patchy
amnesia for the episode
TREATMENT
 Benzodiazepines (Chlordiazepoxide [Librium], diazepam or
lorazepam[Ativan]) should be given in sufficient doses to keep the patient
calm and lightly sedated, then tapered down slowly.
 Carbamazepine or valproic acid can be used in mild withdrawal.
 Antipsychotics (be careful of lowering seizure threshold) and temporary
restraints for severe agitation.
 Thiamine (vitamin B1), folic acid and a multivitamin to treat nutritional
deficiencies (“banana bag”).
 Electrolyte and fluid abnormalities must be corrected.
 Monitor withdrawal signs and symptoms with Clinical Institute Withdrawal
Assessment (CIWA) scale.
 Careful attention must be given to level of consciousness and the possibility
of trauma should be investigated.
 Check for signs of hepatic failure (e.g. ascites, jaundice, caput medusa,
coagulopathy).
 Seizure management:
 Admission
 Ensure safety of room
 Dimly lit and well spread lighting

103
 Use benzodiazepines- Clordiazepoxide 100mg-150 mg/ day, Diazepam 50-
100mg per day orally
o Titrate benzodiazepine to not more than 14 days
 Parenteral Vitamins-Vitamin B complex
 Haloperidol 10mg/ day
 Balance electrolytes and monitor sugar
 On recovery consider counselling/relative/family etc.
 Long term management

A 42-year-old man has routine surgery for a knee injury. After 72 hours in
the hospital he becomes anxious, flushed, diaphoretic, hypertensive and
tachycardic
What most likely can account for this patient’s symptoms?
 Alcohol withdrawal
Treatment?
 Benzodiazepine taper (Chlordiazepoxide [Librium] or Lorazepam
[Ativan] are considered the drugs of choice)
What are you most concerned about?
 Seizures, delirium, hypertension and arrhythmias.

MEDICAL COMPLICATIONS OF ALCOHOL


PERIPHERAL NEUROPATHY
 Often with absence of ankle reflexes.
 Asses and manage accordingly
 Alcoholic myopathy

METABOLIC COMPLICATIONS
 Include hypoglycemia, alcoholic stupor, acute renal failure.
 Always screen for RFT/DM
 Give dextrose intravenously
 Consider diuresis if completely deranged renal function tests (RFT) with loss
of consciousness.

104
WERNICKE’S ENCEPHALOPATHY
 An acute encephalopathy presenting with delirium (confusion), ataxia,
nystagmus and opthalmoplegia occurring in the severely alcohol dependent
usually in the context of withdrawal.
 Patients dying show hemorrhage in brain stem and thalamus.
 These are associated changes in the peri-aqueduct and ventricular areas.
 It is due to thiamine deficiency (Vitamin B1) and requires urgent treatment.
 It may progress to Korsakoff’s syndrome, which presents with cognitive
deficits and confabulation.
 Management:
 Admit
 Withdraw alcohol
 Parenteral thiamine urgent- Pabrinex 2-4 ampoules TDS for 3 days then
daily for 3 days
 Dilute each ampoule in 50-100mls of NS or DNS of 5% and given in 30
minutes.
 Watch for complications- anaphylactic shock, delirium tremens.
 NB: it is importance to give Pabrinex parenterally to malnourished
alcoholics for 3 days once or twice even to those that have vomiting and
diarrhea or intercurrent illness or those with peripheral neuropathy or those
undergoing detoxification.
 Disturbance of consciousness in alcohol can be caused by traumatic
subdural hematoma, hypoglycemia, hepatic encephalopathy and dementia.
They will have initial incontinence, generalized weakness, slurred speech,
ataxia, cerebellar degeneration (gait and stance ataxia).
GASTROINTESTINAL COMPLICATIONS
 Liver disease- obesity, fatty liver, and cirrhosis
 Alcoholic liver cirrhosis and its associated complications (bleeding disorders)
SEXUAL IMPAIRMENT
 Increased arousal
 Impaired erectile function pharmacologically
 Anxiety after impairment ensure decrease in erection
 Direct toxicity on leydig cells of testis, decrease testosterone, decreased
spermatogenesis, infertility, testicular atrophy.

105
FOETAL ALCOHOL SYNDROME
 Heavy drinkers have spontaneous abortion.
 Intrauterine growth restriction (IUGR).
 Foetal alcohol syndrome:
 Developmental and growth restriction/retardation
 Facial and neurological abnormalities
 Brain development impairment
 Behavioral difficulties
 NB: antenatal should screen for alcohol use i.e. use of 1-2 drinks per week.

ASSOCIATED PSYCHIATRIC DISORDERS OF ALCOHOL


ALCOHOLIC HALLUCINOSIS
 In alcoholic hallucinosis, a heavy drinker experiences recurrent auditory
hallucinations, usually of a threatening or derogatory natures.
 The hallucinations occur in clear consciousness.
 Auditory hallucination- unpleasant remarks, commands, persecutory
 The syndrome is an example of a drug-induced psychosis.
 About 10% of people who are alcohol dependent commit suicide.
 Management:
 Assessment
 Admit/or not
 Withdraw alcohol
 Antipsychotics
 Other considerations
PATHOLOGICAL JEALOUS (OTHELLO’S SYNDROME)
 Firm delusions of infidelity.
 Comes from alcohol stemmed impotence.
 Spouse grows indifferent from drunken partner.
 There are repeated accusations.
 Aggressive demands of proof of sexual contact with others
 Violence
 Tragedy-murder
 Note: Alcohol is not the only cause of pathological jealous
 Management:
 Counselling to establish depth and possible treatment
 Complete couple separation permanently

106
DEPRESSION AND ANXIETY
 Alcohol releases inhibitions making it easier to express sadness or self-
destruction impulse.
 The problems that were overcome/overshadowed during drinking suddenly
come back especially during hangover.
 Depression due to alcohol is more common in females.
 It is important not to make diagnosis of anxiety in an alcoholic consuming
person until at least 3 weeks of abstinence since alcohol withdrawal can mimic
anxiety.
 Manage the anxiety or depression and alcohol use.

107
REVISION
CASE STUDIES
CASE ONE: MR JAMES MPUNDU A RETIRED SOLDIER
Mr James Mpundu male and by his NRC which he carries is 70 years old, a well-
known resident of Kabanana, who has been living in a partially completed
building on the allocated plots of Kabanana.
He is loved by his community for his generosity and good cheer. He usually
dishes out money to whoever asks without counting. He loves singing and would
tell loudly stories of the war of 1945 when he was a young man. He would relish
his story especially to the youth and accompanied by small bottles of scotch
whiskey which he produces from pockets of his green army jacket.
The neighborhood had no knowledge of Mr Mpuundu’s visitors, neither did
anyone know where he came from, he does not talk about that. Since you live in
his neighbourhood, one day you meet him as he sat with his usual youthful friends
and you decide to join in their social chat. He welcomes you as he stares directly
at you and offers you where to sit.
‘Oh welcome great sir, do you also live here in Kabanana?’
And you respond, ‘yes sir’
He continues, ‘you know I don’t think my real name is James but that is what is
written on this document, I look at this document, I don’t think that’s my face.
Do you want something to drink, Whisky?’ You answered, ‘Whisky would be
fine’
He fetched a small bottle from his jacket and offered it to you.
He continues, ‘you know I can’t find sleep until I have found enough of these
bottles, previously I would sleep with just one, now I take four to sleep, at some
point I took too much, maybe six, I passed out, when I looked, I was in hospital,
my neighbor was there for me’. ‘I would do anything to find a good beer.’ I also
noticed of late my urine color had gotten deep yellow and I have difficulties
finding myself in the morning, my hands shake but the moment I drink one bottle,
all is well.”
I have had bad dreams of late and even when am alone, the same dreams and
things happen, a man approaches me and asks me to follow him to heaven, he
tells me he is my uncle, do you believe in heaven sir?”
‘I forgot, I usually see the war, it is like its living, am still in it, sometimes in
broad day light, when I recollect myself, and I shake like a leaf.’

108
1. List at least four problems you have identified in James (4 marks)
Answers:
 Alcohol dependence and tolerance
 Post-traumatic stress disorder
 Dementia
 Delusional disorder
 Alcoholic hallucinosis
 Insomnia
 Liver disease (medical disease)
2. Support your listing of each problem identified (4 marks)
Answer:
 Alcohol dependence and tolerance:
o Alcohol dependence: he can’t do without the alcohol; he has tremors of
the hands in the morning which disappear with drinking one bottle. He
probably has alcohol liver disease.
o Alcohol tolerance: he has been increasing the dose, he would previous
sleep when he just drinks one bottle but progressively needed more i.e.
4 and 6
 Post-traumatic stress: He sees the war, the war is alive and he lives in it.
 Dementia: advancing age after 65, he does not really know his name and
can’t recognize his own NRC.
 Delusional disorder: he dreams and sees a person and believes in this
person asking him to go to heaven.
 Alcoholic hallucinosis: when alone he sees a person asking him to follow
to heaven and he hears what he says
 Insomnia: can’t sleep without drinking
 Liver disease: his urine had changed color

3. If you were to listen to his stories any further, what questions would you ask?
(2 marks)
Answer:
 Proper demographic data, next of keen, any previous address
 Personal history- to know who he is
 Family history to know where his family is

109
4. If you were to examine James, what would you be looking for? (5 marks)
Answer:
 General examination:
o Drinking nose
o Jaundice
o Telangiectasia
o Tremors
o Injury marks, neck wrist
o Heart sounds
 Psychiatric examination
o Mental state examination
o Mini-mental state examination

5. How do you want to help James? (5 marks)


Answer:
 Get him to a hospital and get him Admitted
 Withdraw alcohol and detox
 Order investigations:
o Serum: blood glucose (to rule out hypoglycemia), full blood count (to
rule out anemia and any infection), Urea and electrolytes (to rule out
metabolic derangement), Liver function test (to rule out alcoholic liver
disease), triglycerides and clotting profile (to rule out bleeding
disorders), RPR (for syphilis), HIV test
o Imaging: MRI/CT scan to rule out space occupying lessons or other
causes of dementia, Ultrasound and abdominal X-ray to rule out liver
pathology, Chest X-ray to rule out pulmonary infection
o Urine to screen for other drugs
 Establish where his family is
 Pharmacological therapy:
o Give parenteral thiamine, folate and multivitamin (especially B
complex)
o Benzodiazepines.
o Give fluids and monitor blood sugar
o Give Low dose antipsychotic and antidepressant sertraline to manage
anxiety symptoms, PTSD and manic episode symptoms that ensure
o Give naltrexone
o Consider disulfiram when breath alcohol is zero
 Non-pharmacological therapy:
o Psychotherapy: Cognitive behavioral therapy to change his way of
thinking and behavior as well as help him with coping

110
o Reunite him with family for further care
o Regular reviews
CASE TWO: MARY KAMUKWAMBA 30-YEAR-OLD LADY
Mary Kamukwamba, female estimated 30 years presented to your clinic one day
looking worried but could not say what she was worried about.
On direct inquiry, she tells you that she was pregnant and worried about the baby.
You want to ask more questions but she is not willing and looking down, not
talking anymore.
Suddenly she begins to cry and saying it was better if she had died, not to live
like this.
You beg her for an examination and hesitantly gets on the examination bed.
You notice that she had no recollection where she stays and what date that was.
You notice scratch marks around the neck and that she was of bad odour, her hair
initially wrapped in head dress was exposed and had mud and twigs in it.
She trembled a little as you see spots of healed abscesses on her gluteal regions
and upper limbs, some were fresh with central areas that looked like tiny insect
bite sites. These sites where linear, more like following contours of veins.
The abdomen showed a height of fundus of 28cm and there were palpable moving
fetal parts intrauterine.
1. Highlight on relevant history what you want known and helpful towards your
management (3 marks)
Answer:
 Demographic details to know where the patient is from and if of fixed
habitat/house
 Personal history, how this person has grown up and from where
 Family history, where is this persons’ keen, are they alive or dead
 Substance misuse- what type and how often
 Risk for self-harm and harm to others

2. What are the problems that Mary has? (3 marks)


Answer:
 Depression
 Substance abuse
 Risk of self-harm
 Infection
 Pregnancy

111
3. Support your answers in 2 above (3 marks)
Answer:
 Depression: can’t keep eye contact, posture, not volunteering to talk,
wishing her death
 Substance abuse: abscesses, needle track marks
 Risk of self-harm: injuries in wrist and neck
 Infection: abscesses
 Pregnancy: HOF 28cm, and she says she is pregnant

4. You decide to order investigations, restrict yourself to 5 key investigations


since you are in a resource constrained area to give you a window into
management. Highlight why you think such investigations are important (5
marks)
Answer:
 Sepsis screen- HIV/ RPR/HepB
 Full blood count- in view of pregnancy
 Ultrasound for the pregnancy
 Urine drug screen especially for opioids in view of substance abuse needle
track marks
 Liver function test/ renal function tests in view of pregnancy, and infection

5. What would you want to do for Mary before the child is born? (1 mark)
Answer:
 Admit
 Detoxify
 Substitution therapy/methadone

6. Indeed, you have done a good job, Mary safely delivers a baby boy of weight
1.9kg at term. This baby had a high pitched cry, smacking the lips and boxing
occasionally and was failing to brestfeed.
a. What problems do you think the baby has? (2 marks)
Answer:
 Low birth weight
 Withdrawal symptoms
 Feeding problems
 Fluid balance
 Infection

112
b. How do you want to manage this child? (2 marks)
Answer:
 Feeds/EBM
 Replacement therapy- short acting substitution drugs
 Hydration
 Screen for infection

7. How do you want to care for Mary and the child going forward in life? (1
mark)
Answer:
 Long term follow up
 Social welfare care
 Uniting with family
 Continued CBT
 Community reintegration if they had no relative
 Skills training
CASE THREE: A 70-YEAR-OLD RETIRED POLICE OFFICER
Mr. Raman Bwalya was a 70-year-old retired Police officer who was living with
his son and daughter-in-law in Lilayi Police compound. His wife had died some
10 years previously due to an illness. Over the past few years, Mr. Raman Bwalya
had become increasingly forgetful, something his family passed off as ‘just
growing old’. However, the forgetfulness kept getting worse, until one day he lost
his way around his own home. He started forgetting the names of his relatives,
including his favorite grandchildren.
His behavior become unpredictable, on some days he would be irritable and easily
lose his temper, while on others he would sit for hours without saying a thing.
Mr. Raman Bwalya’s physical health began to deteriorate and one day he had a
fit. His son brought him to UTH hospital, where a special scan of the brain was
done, whit showed changes in the structure of the brain which confirmed the
diagnosis of Mr. Bwalya. Mr Raman Bwalya was suffering from a kind of brain
disease commonly found in advanced ages. This illness begins with forgetfulness.
It continues to get worse at time passes and leads to behavioral problems.
1. List 3 important differential diagnosis for Mr. Bwalya (3 marks)
Answer:
 Dementia: Alzeheimer’s/hungtinton/Vascular dementia/ Lewy body
 Space occupying lesion
 Brain Atrophy
 Old age

113
 Infection

2. Explain the differential diagnoses above in 1. (3 marks)


Answer:
 Dementia: forgetfulness, lost his way, forgets relatives, changes in brain
structure, fitting
 Space occupying lesion: radiological changes in brain structure
 Brain Atrophy: brain changes in structure
 Old age: old age

3. What do you think is the definitive diagnosis of Mr. Bwalya’s condition? (1


mark)
Answer: Dementia 2o Alzheimer’s disease

4. What are the pathological changes in this group of illnesses? (4 marks)


Answer:
 Senile plaques
 Tau proteins
 Atrophy of brain
 Gliosis/neuronal loss/ degeneration

5. What line of management would you arrange for him? (7 marks)


Answer:
 Admit
 Investigations
o Sepsis screen: HIV/RPR
o FBC/ESR to rule out infective causes e.g. TB
o RFT/LFT to check for integrity of organs
 Pharmacological therapy:
o Oral cholinesterase inhibitors (Rivastigmine) for mild to moderate
Alzehimer’s based on cognitive assessment score
o Memantine (glutamate receptor antagonist) for severe cases
 Non-pharmacological management:
o CBT:
 Coping with day to day/illness/ old age
 Assistance with remembering
 Cognition: changing and restructuring thoughts
 Behavior: changing behavior

114
CASE FOUR: REGINA MONZE 24 YEAR-OLD WOMAN
I am Regina Moonze, a student at Rusangu University in Monze Southern
Province: “It was so frightening when my situation first happened. I was sitting
on a bus, when all of a sudden my heart started beating so fast that I felt I was
having a heart attack. I had difficulty breathing, and then I started feeling as if
ants were crawling on my hands and feet. My heart started pounding even faster,
my body felt hot and I was trembling all over. I just had to get off the bus, but it
was moving fast and I began to choke.
My biggest fear was that I might collapse or go mad. Then the bus came to a stop
and I rushed to get off even though I was still far from home. Since then I have
never been able to get on a bus makes me feel sick. For the past two years, I have
stopped going out of the house because of this fear and now I have few friends
and almost no social life… I didn’t know what to do and I was too scared to see
a psychiatrist at Chainama Hospital or UTH clinic 6… After all, I am not a mental
case.”
1. What is the diagnosis of the problem of Regina Moonze? (1 mark)
Answer: Anxiety disorder/panic attack/panic disorder

2. Explain why the diagnosis above is definitive (1 mark)


Answer: Patient was not in any eminent danger however perceived the bus to
be dangerous. The patient had the following physical symptoms heart
pounding, trembling, hot body, chocking and psychological changes such as
fear to collapse and fear to go mad.

3. How can Regina be helped? (18 marks)


Answer:
 Collected a detailed history
 Physical examination and Mini-mental state examination (look out for
stigmata pointing towards thyroid disease, previous myocardial or cardiac
disease)
 Investigations (to rule out medical conditions of anxiety):
o Serum: Thyroid function tests i.e. T3, T4 and TSH to rule out thyroid
disease (hyperthyroidism), Full blood count (to rule out anemia), blood
sugar (to rule of hypoglycemia and diabetes mellitus), Serum cortisol
(to rule out Cushing’s disease), RPR for syphilis, HIV test
o Urine for drugs and alcohol (these can present with anxiety like
symptoms and have to be ruled out)

115
o Imaging: MRI/ CT scan to rule out any space occupying lesion or brain
pathology, Chest X-ray (to rule out bronchogenic carcinoma-
paraneoplastic syndrome)
 Non-Pharmacological therapy:
o Psychotherapy: Cognitive behavioral therapy (systematic
desensitization)
 Regina learning and identifying anxiety symptoms and panic
attacks
 Monitor panic attacks using a diary
 Breath and relaxation techniques
 Change in beliefs about panic attacks and phobia to leave the
house
 Exposure of the patient to certain situations that provoke panic
attacks
o Exposure therapy and desensitization
o Cognitive therapy- be ware and replace with realistic thoughts
o Hypnotherapy
o Motivation
 Pharmacological therapy
o Selective serotonin reuptake inhibitors: e.g. Sertraline.
o Mood stabilizer e.g. Lithium
o Atypical antipsychotic e.g. clozapine

CASE FIVE: MABVUTO BANDA, 23 YEAR-OLD YOUNG MAN


OF MUTENDERE COMPOUND
I am Mabvuto Banda of Mutendere compound, Lusaka. I am a Bus conductor
from the time I dropped out of the college in Chalimbana. “I was only 17 years
old when I first started hearing the voices. At first, I wasn’t sure whether they
were in my mind or real. But later, I used to hear strangers talking about me,
saying nasty things. Once I heard a voice telling me to jump into a well and for
days I would stand near the well feeling that I should obey the voice.
I used to feel that my thoughts were being controlled by the TV and sometimes,
I was sure that my food was being poisoned and that gangsters were out to kill
me. I used to get angry and it was when I lost my temper so badly and hit my
neighbor that I was taken to Chainama Hospital.”

116
1. List the lines of interest that would help with diagnosis from this passage. (5
marks)
Answer:
 “I was only 17 years old when I first started hearing the voices”
 “At first, I wasn’t sure whether they were in my mind or real. But later, I
used to hear strangers talking about me, saying nasty things.”
 “Once I heard a voice telling me to jump into a well and for days I would
stand near the well feeling that I should obey the voice.”
 “I used to feel that my thoughts were being controlled by the TV”
 “Sometimes I was sure that my food was being poisoned and that gangsters
were out to kill me”
 “I used to get angry and it was then I lost my temper so badly and hit my
neighbor”

2. At this age what do you think is the diagnosis of Mr. Banda? (1 mark)
Answer: Paranoid Schizophrenia

3. What investigations would you do for Mr. Banda? (4 marks)


Answer:
 Blood investigations
o Endocrine: thyroid function tests (hyper/hypothyroidism), serum
calcium (hyper/hypocalcemia)
o Infections: RPR for syphilis, HIV test, Hep B
o Supportive: Full blood count, U and Es and LFTs (For management of
drugs mostly)
 Urine drug screen: to rule out substance abuse and withdrawal (e.g.
cannabis, benzodiazepines, barbiturates, PCP, inhalants, alcohol,
hallucinogens, ecstasy)
 Imaging
o MRI/CT: to rule out CNS disease (Multiple sclerosis, tumors and space
occupying lesions)
o Chest X-ray

4. What treatment plan would work for him? (10 marks)


Answer:
 Non-pharmacological therapy: Psychotherapy
o Cognitive behavioral therapy: up to 16 sessions, focusing on re-
evaluating abnormal thoughts and perceptions and reducing the distress
resulting symptoms

117
o Drug use advice
o Stoppage/replacement
o Skill training- young man
 Pharmacological therapy:
o Antipsychotics (typical) e.g. haloperidol for violet episodes
o Antipsychotics (atypical): e.g. Clozapine
o If no improvement is noted a different drug can be used or combination
therapy
o Long term therapy

CASE SIX: LINDA MWANGO 43-YEAR-OLD MIDDLE AGED


WOMAN
My name is Linda Mwango and I live in Chilenje South, Lusaka. My situation
was as follows: “It started quite gradually, but before I knew it I had lost all
interest in my life. Even my children and family didn’t make me feel happy. I was
tired all the time. I could not sleep, I used to wake up 2am or 3am in the morning
and then just toss and turn. I lost the taste for food which I used to love and I lost
weight too.
I even lost interested in reading because I just could not concentrate. My head
ached. I felt so lousy about myself that I was a burden on the family and so on.
The worst thing was that I felt embarrassed about the way I felt and could not tell
anyone. My mother-in-law used to complain that I had become lazy. Once I felt
like ending my life and it was there that I got so scared that I told my husband…
That was two months after I started feeling ill.”

1. List any three differential diagnoses for Mrs. Mwango (3 marks)


Answer:
 Depression
 Delirium- hypo alert hypo active
 Sleep disorder

2. Support your differential diagnoses listed above (6 marks)


Answer:
 Major depressive disorder: loss of interest in life, loss of interest in hobbies,
fatigue, insomnia, weight loss, inability to concentrate, somatic symptoms
(headache), suicidal ideation.
 Delirium- head ache, two months after illness
 Sleep disorder- was awake all the time and sleep left

118
3. What is the definitive diagnosis for Mrs. Mwango? (1 mark)
Answer: Major depressive disorder

4. How would you manage her? (10 marks)


Answer:
 Detailed history (both objective and subjective history)
 Physical examination with mini-mental state examination
 Investigations (to rule out any underlying conditions)
o Serum/ blood: HIV test, RPR for syphilis, FBC (for anemia), thyroid
function tests for hypothyroidism. Supportive tests such as LFTs, U and
Es before medication and be commenced.
o Urine for substance abuse
 Pharmacological therapy
o Antidepressants: Selective Serotonin reuptake inhibitors: Selective
serotonin reuptake inhibitors: Sertaline
o Antipsychotic: e.g. clozapine
 Non-pharmacological therapy
o Psychotherapy
 Cognitive behavior therapy (CBT) to help her realized and
recognize distorted thinking and then change behaviors and
thinking.
 Counselling to explain illness
 Find cause and deal with cause
 Electroconvulsive therapy if she does not respond to above treatment

CASE SEVERN: A 38-YEAR-OLD YOUNG ADULT MAN


I am James Phiri living in Roma Township along Mutandwa Road in Lusaka, “I
used to feel as if I had so much energy that I did not need to sleep at all. In fact, I
hardly slept in those days. I would rush about with all my schemes and plans, but
never really managed to finish any of them properly. I used to lose my temper if
anyone tried to stop me. Once I got into a big fight with my business partners
over one of my crazy schemes. But when I was high, I never realized how wrong
I was. I even felt sometimes that I had special powers to heal others. The worst
thing about my illness was how I spend so much money that I almost bankrupted
the family. “I was boastful, and never settled in one place and further I would
change topic to topic.”

119
1. What is the problem with Mr. Phiri? (2 marks)
Answer:
 Hyperactivity- mania
 Getting annoyed easily- Paranoid
2. List 2 other differential diagnoses (2 marks)
Answer:
 Mania
 Bipolar I disorder

3. How would you plan his management? (16 marks)


Answer:
 Detailed history and physical examination including mini-mental state
examination
 Investigations:
o Urine for drugs screen
o Blood/Serum: Thyroid function test (for hyperthyroidism), FBC, RPR
for syphilis, HIV test, U and Es, LFTs and creatinine.
o Imaging: ECG
 Pharmacotherapy:
o Lithium is a mood stabilizer. Watch out for side-effects
o The anticonvulsants carbamazepine and valproic acid are alternative
mood stabilizers.
o Atypical antipsychotics (risperidone, olanzapine, quetiapine,
ziprasidone) are effective as both monotherapy and adjunct therapy for
acute mania. A combination of a mood stabilizer and antipsychotic may
be most effective.
 Non-pharmacological therapy (Psychotherapy):
o Cognitive behavioral therapy
 Cognitive- a way of thinking change
 Behavior- changing behavior
o Individual therapy
o Community integration
o Social skills

120
CASE EIGHT: A 38 YEAR-OLD DIVORCED MOTHER OF
TWO TEENAGERS
Chimuka Munkombwe is a 38 year-old divorced mother with two teenagers.
She has had a successful, well praying occupation in Ndola of the past several
years in top-level management. Even though she has worked for the same,
growing company in Ndola for over 6 years, she has found herself worrying
constantly about losing her job and being unable to provide for her children
despite no company threats.
This worry has been troubling her for the past 8 months. Despite her best
efforts, she hasn’t been herself feeling restless, tired and tense. She often paces
in her office when she’s there alone. She’s had several embarrassing moments
in meetings where she has lost track of what she was trying to say.
When she goes to bed at night, it’s as if her brain won’t shut off. She finds
herself mentally disturbed and in the worst case scenarios regarding losing her
job, including ending up homeless.
1. What is your diagnostic formulation? (4 marks)
Answer: Chimuka who is 38 years old has 2 teenage children, has a good job
and worried of losing it all the time. She gets restless, tired and tense and
would not concentrate at work. She cannot sleep well at night. She has
anxiety disorder most likely Generalized anxiety disorder.

2. What will be your management plan? (16 marks)


Answer:
 Detailed history and examination:
o Looking at when this problem started
o What exacerbates the problem
o What reliefs it
o Personal history to highlight source of fear or anxiety
 Investigations
o Blood: FBC, RFTs/LFT, Thyroid function tests
o Urine for drug screen
o Imaging: ECG, Chest X-ray, CT/MRI scan of head
 Psychotherapy
o Cognitive behavioral therapy to change cognitions (thoughts) by
learning about the anxiety and behaviors
o Self-coping mechanisms
o Relaxation techniques
o Breathing techniques

121
 Pharmacological therapy
o Anxiolytic e.g. benzodiazepines
o Antidepressants e.g. SSRIs sertraline
o Antipsychotics e.g. clozapine (Atypical)
CASE NINE: A 27 YEAR-OLD YOUNG MALE ADULT
Johanis Milimo is a 27-year-old male living in Avondale Lusaka who recently
moved back in with his parents after his fiancée was killed by a drunk driver 3
months ago. His fiancée, a beautiful young woman he had been dating for the past
4 years, was walking across a busy road intersection in Cairo Road of Lusaka to
meet him for lunch one day. He still vividly remembers the horrific scene as the
drunk driver ran the red light, hitting down his fiancée right before his eyes.
He raced to her side, embracing her crumpled, bloody body as she died in his
arms in the middle of the crosswalk. No matter how hard he tries to forget, he
frequently finds himself reliving the entire incident as if it was happening all over.
Since the accident, Milimo has been plagued with nightmares about the accident
almost every night. He had to quit his job because his office was located in the
building right next to the little café where he was meeting his fiancée for lunch
the day she died. The few times he attempted to return to work were unbearable
for him. He has since avoided that entire area of the Lusaka Town.
Normally an outgoing, fun-loving gentleman, Milimo has become increasingly
withdrawn, “jumpy”, and irritable since his fiancé’s death. He has stopped
working out, playing his guitar, or playing basketball with his friends – all
activities he once really enjoyed. His parents worry about how detached and
emotionally flat he has become.
1. What are the two differential diagnoses? (2 marks)
Answer:
 Posttraumatic stress disorder
 Depression

2. Explain your differential diagnoses (2 marks)


Answer:
 Posttraumatic stress disorder: witnessed horrific accident, relives it all the
time
 Depression: an outgoing person has stopped all activities of interest and is
withdrawn

122
3. Are these diagnoses separate from each other or are existing together?
Explain your answer (2 marks)
Answer: they are existing together as they tend to occur comorbid and it is
hard to distinguish where one ends and where the other starts. However, a
diagnosis can be made.

4. How do you manage Milimo? (14 marks)


Answer:
 Investigations
o Urine for drug screen
o Serum/blood: FBCs, LFT/RFT, TFT
 Psychotherapy
o Trauma focused CBT
o Address Education/guilt/shame caused by accident
o Imagery replacement
o Exposure
o Cognition
o Modification
o Eye movement desensitization and reprocessing (EMDR): eye
movements while focusing on the memory
 Pharmacological
o SSRIs e.g. sertraline
o Atypical antipsychotics e.g. Clozapine

CASE TEN: A 55 YEAR FEMALE PRIMARY SCHOOL


TEACHER

Loveness Mbangweta is 55, a Primary School Teacher and mother of seven,


suffered from an illness. Her family believed that her illness was due to evil spirits
and took her to various shrines in Kalabo, but she showed no signs of
improvement. She felt like leaving her home and running away and attempted
suicide several times. One of Loveness Mbangweta’s colleagues told her about a
mental health outreach facility run by the Primary Health Care Team in Kalabo.
There, psychiatric personnel diagnosed a severe illness.

123
Loveness Mbangweta started attending counseling sessions and taking
medication. In six months, her condition started to improve. After three years of
medication and regular follow up from the Primary Health Care Team, the doctor
advised that she could stop her medication. Since 2016, Loveness Mbangweta
has been working at the Women’s Group Development project, launched by PHC
Team, in Kalabo where she trains and assists women recovering from mental
illness and their caregivers to learn a trade and develop small businesses.
1. What psychiatry illness did the family to loveness suspect? (3 marks)
Answer:
 Schizophrenia
 Depression
 Delusional disorder

2. Would you explain why you have listed the illnesses in 1 above? (3 marks)
Answer:
 Schizophrenia: running away
 Depression: the illness was unknown, family classified it as evil spirits
 Delusional disorder: the family believed in evil spirits

3. What is the diagnosis? (1 mark)


Answer: any of the three

4. What is your comment about the current engagement by the PHC team after
discharge from Hospital? (2 marks)
Answer:
 Adequate engagement, the way it should be with outreach
 Patient made active than idleness
 A form of therapy to look after other people

5. Would you have managed loveness differently? Explain your answer and
alternatives you could have taken (8 marks)
Answer:
 Yes
 Detailed history and examination is needed:
o To look for presentation
o To enable narrow of differential diagnosis
o To understand family beliefs/sufferer
o Culture
o Examine Mental state

124
 Investigations
o Blood: FBC, RFTs/LFT, Thyroid function tests
o Urine for drug screen
o Imaging: ECG, Chest X-ray, CT/MRI scan of head
 Psychotherapy
o Cognitive behavioral therapy: to help change thoughts (cognitive) and
behaviors as well as cope with illness
 Pharmacotherapy
o Antipsychotics e.g. clozapine for schizophrenia and delusions
o Antidepressants e.g. SSRIs Sertraline for depression
o Mood stabilizer e.g. lithium

6. The family to loveness took her to shrine, of what psychiatry significance is


this history? (3 marks)
Answer:
The family took loveness to the shrine is significant because it gives an
indication of cultural beliefs of the family, family history and social status

125

You might also like